Kaplan QuantPart0001 PDF

You might also like

Download as pdf
Download as pdf
You are on page 1of 142
Quantitative Reasoning va arecentr et (nnn ay evn aaa omossus: Le. (S). ne eee ae ees Senne teeten ot ctogetnbdneee a Revised GRE’S Gcvy Iipag 3-3 As GRE Center's oho i - (fF ic Matie Easy For Bangladeshi Graduates Revised GRE -4a >f% eefoa aisetl 42 ETS Revised GRE Official Guide Barron's New GRE Kaplan New GRE McGraw Hills New GRE Princeton Review New GRE And other New GRE Books www.¢ ceriter’ nat (3) Lalmatia: Hosue : $1) #01921-080848 | = 01720-557481 ore... (#) aon Papal Kenia dots40-a42162 Tau Introduction to Quantitative Reasoning OVERVIEW ‘The Quantitative Reasoning section of the GRE is designed to place most of its emphasis on your ability to reason quantitatively—to read a math problem, understand what it's asking, and solve it. The mathematical concepts tested on the GRE are similar to those tested on the SAT. You will see questions related to arithmetic, algebra, geometry, and data interpretation. There is no trigonometry or calculus on the GRE. The emphasis in the Quantitative Reasoning section is on your ability to reason, using your knowledge Of the various topics. The goal is to make the test an accurate indicator of your ability to apply given information, think logically, and draw conclusions. These are skills you will need at the graduate level of study. In this section of the book, we'll take you through all the types of Quantitative Reason- ing questions you'll see on the GRE and give you the strategies you need to answer them quickly and correctly. Also, all of the mathematical concepts you'll encounter on the test are included in the "Math Reference” Appendix at the back of this book. Think of the examples there as building blocks for the questions you will see on the test. QUANTITATIVE REASONING QUESTION TYPES The GRE contains two Quantitative Reasoning sections with approximately 20 ques- tions each, Each section will ast 35 minutes, and be composed of a selection of the following question types: ‘www.grecenter.net 183 e Laman: esue 2, Block A(OppDhaond Govt oye 901931 enna]! ‘2: 01720-557481 ore.. (# sanank Howse 11, Road 4 Block A 001677 481578 euuemmon eae meaner 184 Part Three: Quantitative Reasoning Chapter 9 + Quantitative Comparison + Problem Solving + Data Interpretation The Quantitative Reasoning portion of the GRE draws heavily upon your ability to combine your knowledge of mathematical concepts with your reasoning powers. Specifically, it evaluates your ability to: + compare quantities using reasoning. + solve word problems. + interpret data presented in charts and graphs. Within each section of Quantitative Reasoning questions on the GRE, you will see an assortment of question types. PACING STRATEGY As a multi-stage test, the GRE allows you to move freely within each section, which can be a big advantage on test day. If you get stuck on a particular question, you can tag it and come back to it later when you have time. You only score points for correct answers, so you don’t want to get bogged down on one problem and lose time you could have used to answer several other questions correctly You will have 35 minutes to work on each Quantitative Reasoning section. The 20 questions in each section will be an assortment of Quantitative Comparison, Problem Solving, and Data Interpretation items. However, these types are not distrib uted equally. The chart below shows how many questions you can expect of each ques- tion type, as well as the average amount of time you should spend per question type. Quantitative Problem Data a) “Comparison _Solving Interpretation Number of Questions approx. 10 approx. 7 ‘approx. 3 Time per Question _1.5 minutes 2 minutes 2 minutes www.grecenter.net grecenter@gmail.com Lalmati: Horwe 2/1, lock (Opp Dhanmondi Govt Boy! #01921-080648 =: 01720-557481 ‘ore, Settee toed Kula: Rekib Properties, House 11,Senadanga Main Rd #01940-442162 lock A 401677-481978 Part Thre: Quantitative Reasoning Introduction to Quantitative Reasoning | 185 Try to keep these time estimates in mind as you prepare for the test. If you use them as you work the practice items, you will be comfortable keeping to the same amounts of time on test day. Additionally, you will be prepared to use the Mark and Review buttons to your advantage while taking the actual test. To CALCULATE OR NOT ‘An on-screen calculator will be available during the GRE, but there are several points to consider about its use. A calculator can be a time saver, and time is immensely impor- tant on a standardized test. But while calculators can speed up computations, they can also foster dependence, making it hard for you to spot the shortcuts in GRE questions. Using the calculator for a long, involved computation to answer a question will gobble up your allotted time for that question—and perhaps for several more. You may even. make a mistake in your computation, leading to an incorrect answer. Remember, tl is a reasoning test. The quantitative questions on the GRE are not designed to require lengthy computations. If that is the case, why is a calculator provided? A calculator can be an asset for the occasional computation that a few questions require. It may prevent an error caused by a freehand calculation. The on-screen calculator provided is a simple, four-function calculator. An image of the calculator is provided below, showing the function keys, including the square root key and change of sign key. By not relying on the calculator, you will be free to focus on interpreting numbers and data and using your critical thinking skills. This is the intention of the writers of the test. For example, Problem Solving questions will likely involve more algebra, and Quantitative Comparison questions will require more reasoning than calculating. ‘www.grecenter.net Lalmatia: Hosue 2/1, Block A (Opp Dhanmondi Govt Boys’) #01921-080848 = 01720-57481 gre. Banani Mowe 12, Road Bleck A A017 481578 grecenter@gmail.com hula: Reb Properties, House 11,Sonadanga Maln Rd w01940-442162 Patt Thee: Quantitative Ressoning 186 | Chapters NAVIGATING THE QUANTITATIVE FOUNDATIONS AND CONTENT REVIEW SECTION The chapter immediately following this one concems Quantitative Foundations and Content Review and will review the classic math concepts and topics that you may encounter on the GRE. This section of the book also includes individual chapters on Quantitative Comparison, Problem Solving, and Data Interpretation questions. Each chapter includes an introduction and definition of the relevant question types, and a review and examples of the strategies to follow to answer those questions quickly and correctly. In addition, youll find a practice set of questions with answers and explana- tions for each of the question types you'll encounter on the GRE. Finally, at the end of this section, you'll find the Quantitative Reasoning Practice Sets, two sets of 60 Quantitative Reasoning questions along with answers and explanations. Use the Quantitative Practice Sets to test your skills and pinpoint areas for more focused study. When you are finished with this section of the book, you should be thoroughly prepared for any question you might encounter on the Quantitative Reasoning sec- tion of the GRE. Lalmati: Hosue 2/1 Block A (Opp Dhanmondi Govt Boys) #01921-000848| ‘Banani Houee 121,Road 4 lock A #01677-481878) ‘Khulna: Rak Properties, House 11,Senadanga Main Rd #01940-442162| grecenter@gmail.com Urata wey Quantitative Foundations and Content Review INTRODUCTION TO QUANTITATIVE FOUNDATIONS AND CONTENT REVIEW The GRE tests a variety of math concepts that fall outside the range of straightforward algebra and arithmetic questions. These classic concepts are the foundation on which ‘many other types of GRE questions are built. They include: + Percentages + Simultaneous Equations + Symbolism + Special Triangles + Multiple and Oddball Figures + Mean, Median, Mode, and Range + Probability This chapter will cover all these concepts, including specific strategies to conquer ques- tions pertaining to each one. www.grecenter.net = 01720-557481 grecenter@gmail.com ImataeHesue 2, lock A (Opp Dhanmondi Govt Boye) #01921-000848| ‘Banani: House 121,Road 4 Block A #01677-481878| ulna: Rak Properties, House 11,Sensdanga Main Rd 401940-442162| e 187 Part Three: Quantitative Reasoning 188 | Gna THE KAPLAN APPROACH TO PERCENTAGES A percent is really a comparison of a number to 100; 17% means 17 per 100, for example. Since a percent is a comparison it can also be expressed as a fraction: 17% = =! And since the comparison is to 100, a percent can also be expressed as a decimal: 17% = 0.17. This ease of conversion between fractions, decimals, and percents leads to many uses for Percents and many opportunities for problems involving them. Most percent problems involve three quantities—whole, part, and percent. You are Usually given two of the quantities and asked to find the third. First, recall the basic nature of a percent (a comparison of one number to another) and how to convert that ratio to a decimal and a percent. To find a percent, find the ratio of the part to the whole, and then write that ratio in Percent form. Example: What percent of 60 is 45? = Part Percent = Whole =45 3 Percent = 45 = 3 75% To find a part when you know the percent and the whole, multiply the whole by the decimal or fraction equivalent of the percent. Example: What is 20% of $802 Part = Percent x Whole Part = 20% x $80 = 0.20 x $80 = $16 The question below shows how percentages are applied to real situations. Last year, Julie's startup company showed a profit of $20,000. This year, the same company showed a profit of $25,000. If her company shows the same percent increase in profit in the coming year, what will that profit be? ® $27,500 $30,000 @ $31,250 ® $32,500 ® $35,000 The problem about Julie's company asks us to calculate her profit next year— that Is, her current profit plus an anticipated increase. Her previous increase was $25,000 — $20,000 = $5,000. worwgrecentermet ois oocnen aa gy Manage ace ace =omosn GLE. Sent towne 1, tends Deck for? 4357 Ge mnreoaicoe Khulna Rak Properties, House 11,Sonadangs Main Ré#01940-442162| Part Three: Quantitative Reasoning ‘Quantitative Foundations and Content Review To express that increase as a percent, compare the amount, $5,000, to the previous year’s profit, $20,000. That is, find what percent $5,000 is of $20,000. Part__ $5,000 Whole ~ $20,000 ~ 4 Percent 25% Now you need to find the amount of the anticipated increase in profit for next year—so find 25% of $25,000. Part = Percent x Whole = 25% x $25,000 = 1 x $25,000 = $6,250 Finally, add the amount of the anticipated increase to the current profit to find her next year's profit: $25,000 + $6,250 = $31,250. Choice (€) is correct. AA useful trick that sometimes saves a step is to add or subtract a given percent from 100%. For example, suppose an $80 item is marked down 40%. To find the new price, you can find 40% of $80 and then subtract: 40% X $80 = 0.40 x $80 = $32 and $80 — $32 = $48 Or you can recognize that 100% ~ 40% = 60% and find 60% of $80: 60% X $80 = 0.60 x $80 = $48 As another example, suppose an $80 item is marked up 20%. To find the new price, you can find 20% of $80 and then add: 20% X $80 = 0.20 x $80 = $16 and $80 + $16 = $96 Or you can recognize that 100% + 20% = 120% and find 120% of $80: 120% x $80 = 1.20 x $80 = $96 KAPLAN STRATEGY You can save a lot of time on test day by knowing the percent and decimal equivalents of the following commonly tested fractions. 3 12.5% = 0.125, = 375% = 0.375 3 = 668% = 05 12 165% = 018 ie 3 6 16.6% = 0.16 40% = 0.4 a 12 = 50% = 4 5 20% = 0.2 50% = 0.5 5 4 = 25% = 0.25 60% = 06 2 = 833% = 0.83 $= 333% = 03 = 62.5% = 0.625 3 = 075% = 0.875 ‘www.grecenter.net e Lalmatia: Hosue 2/1, Block A (Opp Dhanmondi Govt Boys’) #01921.080848 =omosomsi OLE, Seutowe htt ae A aetra974 grecenter@gmail.com tuina: Rak Properties, House 11, SonedangaMan Rd #01940-442162 189 Part Three: Quantitative Reasoning 190 | cope PERCENTAGES PRACTICE SET 1. Ifa sweater sells for $48 after a 25% markdown, what was its original price? @D $56 $60 @ sot D 808 @® $72 2. A hardware store is selling a lawnmower for $300. If the store makes a 25% profit on the sale, what is the store's cost for the lawnmower? @® s210 $225 © 8240 @®D $250 ® ss 3. A retailer marked up the cost of a coat by 20% when she first displayed it in her store. After several weeks, she reduced the selling price of the coat by 25%, If the retailer originally paid $50 for the coat, what will be her loss on the coat at the final price? D so $5 © so ®D ss ® 82 worwgrecenternet ents enna pps esau oreoranbea =omosua GLE. Pastis tH bsdaanea totter facmtarOnailcom Khulna: Ral Properties, House 11,Sonadanga Main Rd #01990-442162| Part Three: Quantitative Reasoning (Quantitative Foundations and Content Review | 191 Percentages Practice Set Answers and Explanations follow on the next page. www.grecenter.net 01720-557481 grecenter@gmail.com ‘Lalmtia:Hosue 21, Block A (Opp Dhanmonsl Govt Boys) #01921-080848) Banani: House 121, oad 4, Block A.#01677-481978| Khulna: Rab Properties, Mouse 11,Sonadanga Maln Rd #01940-442162 Part Three: Quantitative Reasoning 192 | Ghnerio PERCENTAGES PRACTICE SET ANSWERS AND EXPLANATIONS 15 The price after a 25% markdown is $48, so $48 is 75% of the original price. Percent x Whole 75% X Original Price 0.75 x Original Price = $48 48 os Original Price Original Price = The original price was $64, which is choice (C). zee The selling price of $300 represents 100% of the store's cost plus 25% of the store's cost. Use 0.25 Cost to represent the profit in an equation; solve for the cost Selling price = Cost + Profit Selling price = 1 x Cost + 0.25 x Cost $300 = 1 X Cost + 0.25 x Cost $300 = 1.25 x Cost $300 _ 25 Cost $240 = Cost ‘The store's cost for the lawnmower was $240, choice (C). 3. B The retailer first set the selling price of the coat at 120% of cost: 1.20 X $50 = $60. Later, she reduced the selling price 25%: 0.75 $60 = $45. She will lose $5 selling the coat at that price. Choice (B) is correct. ‘www.grecenter.net {matin Honus 2, Block (Opp Dhanmondi Gov Boyt #01921 08048 = 01720-557481 Gre. ‘Sana Nowe 2, fou Block A 01677481978 Geataeoaalcom Khulna Ral Properties, House 11,Sonadangs Main Ré#01940-442162| Part Three: Quantitative Reasoning CunttatveountonsandContntferen | 193 THE KAPLAN APPROACH TO SIMULTANEOUS EQUATIONS Solving a one-variable equation means finding the value of the variable that makes the equation true, But what about problems in which there are two equations with the same two variables in each equation? When there are two equations with two variables to be solved so that both equations are true simultaneously (at the same time), then those equations are sometimes called a system of simultaneous equations. Take a look at the simultaneous equations below. Ifp + 2q = Wand 3p + q = 12, then p = @®3 You could solve this problem with a method called substitution. Using that method, you solve one of the equations for one variable in terms of the other, and then substitute the resulting expression for the variable into the other equation. Then you have an equation with just one variable instead of two, and you solve for it, But simultaneous equations on the GRE are often more easily solved by a method called combination, in which you add or subtract equations to eliminate one of the variables. The combination method is shown below. For the simultaneous equations p + 2q = 14 and 3p + q = 12, you cannot eliminate a variable by adding or subtracting the equations in their given form. The coefficients of the respective variables are not opposites of each other. But if you multiply both sides of an equation by a number that makes one set of coefficients opposites, then you have a way to solve the system. Here’s how it works for this system of equations. 1, Write one equation underthe other. 2. ‘Multiply the second equation by —2. p+2q=14 p+2q=14 Bp+q= 2 2p + 9) = ~2(12) www.grecenter.net =omossae Ore. grecenter@gmail.com Lalmatis:Hosue 2/1, lock (Opp Dhanmondi Govt Boy 401921-080848 Banani: House 121, Road 4, Block A.#01677-481976| Kula: Rai Properties, Mouse 11,Sonadanga Main Rd #01940-442162 Part Three: Quantitative Reasoning 194 | chapteri0 3. The sum of +29 and -2q is 0,soqwillbe 4. Solve forp. eliminated when the equations are added. p+2q p+2q=14 + 6p - 29 = 6p - 2q = -24 =5p Choice (D) is correct. Now, if you also need to know the value of q, substitute 2 for p into either original equation. Use the first equation: Or use the second equation: p+2q=14--2+29=14 3p +q=12>32)+q=12 2q= 12 6+q=12 q=6 q=6 In some cases there is no single value for each variable in a system of simultaneous equations. For example, consider this system. 4a +2b=10 12a + 6b = 30 When you try to solve this system, you find out that the equations are equivalent. if you try to eliminate one variable, you end up eliminating both variables. 1, Write one equation under the other. 2. Multiply the first equation by 3. 4a +26 = 10 3(4a + 2b) = 3(10) 12a + 6b = 30 12a + 66 = 30 3. Notice that the equations are equivalent. 4. When the one equation is 12a + 6b = 30 subtracted from the other, both 1204 6b = 30 variables are eliminated and a true statement remains. 1a + 6b = 30 12a + 66 = 30 o=-0 This means that there is no single value for each variable; there are an infinite number of possible sets of values for a and b. wargrecenternet_— = sss isa pe Scoala oon coi =omosoms OLE, nutes tnd greThaar grecenter@yinaiicom ‘Khulna Rak Properties, House 11, Sonadangs Main Ré#01940-442162| Part Three: Quantitative Reasoning ‘Quantitative Foundations and Content Review | 195 SIMULTANEOUS EQUATIONS PRACTICE SET L Ifx+y=8andy—x 2, then y = @-2 3 © s5 Des ® wo 6 10 15 3. Which of the ordered pairs of numbers (c, d) satisfies the simultaneous ‘equations shown? etd 30 — 6d @® (2,4) @® 1-3) @ 0,2) Dav ® B-9 ‘www.grecenter.net e matin: Hosue 2, Block (Opp Dhanmondi Govt Boy} 01921-080848| ‘Banani: House121,Road 4 Block A#01677-481978 01720-557481 grecenter@gmail.com ulna Ral Properties, House 11,Sonadanga Man Rd #01940-442162| Part Three: Quantitative Reasoning 196 | Ghaptrio SIMULTANEOUS EQUATIONS PRACTICE SET ANSWERS AND EXPLANATIONS 1B When you add the two equations, the x’s cancel out and you find that 2y = 6, so y choice (B) 2 Multiply the first equation by 2, and then subtract the first equation from the second to eliminate the ms and find that 5n = 5, or = 1. Plugging this value form into the first equation shows that m = 6, som + n= 7. The correct answer is (C). aA if you multiply the first equation by 3, the result is -3c ~ 6d = ~18, So the two equa tions are equivalent. To find a pair of numbers that satisfies the equations, solve one equation for d. 6 -c+6 ile deta ‘The ordered pair (c, d) that satisfies the relationship is (—2, 4). Choice (A) is correct. ‘www.grecenternet {ama Hose 27, Bleck Opp Dhanmondl Gav Boy) 40192-20048] ‘© 01720-557481 Gre. Danent te 11 tod Dock A got677-45279| ee teg onion \ulnactalb Properties Hoe 1 Sonadanga Mai Rd ¥1340-a42162 Part Three: Quantitative Reas Quantitative Foundations and Content Review 197 THE KAPLAN APPROACH TO SYMBOLISM You expect to see arithmetic symbols such as +, ~, x, +, and 9 on the GRE, but you are likely to also encounter some very unusual symbols. You may be asked, for example, to find the value of 10 # 2, 5 © 7, or 65 ¥ 2. Some GRE questions use unusual symbols as a way of testing your math and abstract reasoning ability, Such questions turn out to be not as difficult as they first appear; you simply need to substitute normal math symbols for the unusual ones and then perform the operations. Ifa b = Va ¥ 0 for all nonnegative numbers a and b, what is the value of 1096? @o @2 @4 @Ds ® is To solve the problem, substitute 10 for a and 6 for b in the expression Va + 6, Va+b =\10F6 = \16 =4 Choice (C) is correct. Now take a look at a more involved symbolism question. Ifa A means to multiply a by 3 and a + means to divide a by —2, what is the value of ((8 ) A) 4? 08608 This question seems to ask a lot, but don't be fooled. If you take it one step at a time, it’s much easier to manage. worw.grecentennet lati Hon, lc (Opp Dbamendl ov Boy) 40193-2004] = omossae1 ‘GT (=) anarouee 12 fond lc A. 1%67749978 0-5 Khulna: Rakib Properties, House 11,Sonadanga Main Rd #01940-442162 grecenter@gmail.com Part Three: Quantitative Reasoning 198 | chapter10 To solve this equation, we need to remember the order of operations. When an expres- sion contains a set of parentheses nested inside another set of parentheses, begin with the innermost set first. The question stem tells us that a & means to divide a by ~2, s0 8 means to divide 8 by ~2. ae So replace (8 *) in the expression with —4, and the expression is now (—4 a) And 4 & means to multiply ~4 by 3. -4&=-4x3=-12 Next, replace (~4 A) with —12; the expression is now —12 *. And —12 + means to divide —12 by ~ 12% 6 2 Even symbolism questions that appear to be difficult are manageable when you solve them one step at a time. Choice (E) is correct. SYMBOLISM PRACTICE SET 1. If x # 0, let # x be defined by # x = x — = Then @ (-3) = 10 ® -¥ ® -§ @ 0 @D é 3 0 ® ® r (7 — 3) for all integers r and s, then 4 (3 ¥ 5) = ‘www.grecenter.net e Lalmatia: Hosue 2/1, Block A (Opp Dhanmondi Govt Boys’) #01921-080648 =omosme GLE. Sunnttowe tend bleckht167748978 satecsepeet oan sentences grecenter@gmail.com Part Three: Quantitative Reasoning Quantitative Foundations and Content Review | 199 3. Ife , where c # 0, what is the value of 12 008 6 08 worm grecenternet x>0>y>z A Quantity B wty xtZ ant CB] Quantity A is greater Quantity B is greater. © The two quantities are equal. ®D The relationship cannot be determined from the information given. © STEP1 Analyze the centered information and the quant ies. In this problem, there are four variables: w, x,y, and z. You are asked to compare the values of the sums of pairs of variables. You know the relative values of the different variables, but you don't know the actual amounts, You do know that two of the variables (wand x) must be positive and two of the variables (y and 2) must be negative numbers Latmatia: Hose 211 Block R (Opp Dhanmondi Govt Boys) #01921-080848| ‘Banani Houre 121,Road 4 lock A #01677-481973) ‘Khulna: Rak Properties, House 11,Sensdanga Main Rd #01940-442162| www.grecenternet =omossue ‘GLe grecenter@gmail.com Part Three: Quantitative Reasoning 224 | chapters1 © STEP2 Approach strategically. In this case, think about the different sums as pieces of the whole. if every “piece” in one quantity is greater than a corresponding “piece” in the other quantity, and the only ‘operation involved is addition, then the quantity with the greater individual values will have the greater total value. From the given information, we know that: sw>x yz The first term, w, in Quantity A is greater than the first term, x, in Quantity B. Similarly, the second term, y, in Quantity A is greater than the second term, z, in Quantity B. Because each piece in Quantity A is greater than the corresponding piece in Quantity B, Quantity A must be greater; the answer is (A). Now let’s apply the Kaplan Method to a third Quantitative Comparison question: ‘The diameter of circle O is d, and the area is a. Quantity A Quantity B ad? a 2 D Quantity A is greater. Quantity B is greater. © The two quantities are equal. D The relationship cannot be determined from the information given. © STEP1 ‘Analyze the centered information and the quantiti In this problem, you are given additional information: the sentence that tells you the diameter of circle O is d and the area is a. This is important information, and gives you a key to unlocking this question. Given that information, you can tell that you are comparing the area, a, of circle O and a quantity that includes the diameter of the same circle, If you're thinking about the formula for calculating area given the diameter, you're thinking right! www.grecenter.net e | almatia: Hose 2/1, Block A (Opp Dhanmondi Govt Boys! #01921-080848 =omossun OLE. ‘anaiowe 121, oud 4 block A w01677 401978 grecenter@ymail.com ulna Rakb Properties, House 11, Sonadanga Main Rd #01940-442162| Part Three: Quantitative Reasoning ‘Quantitative Comparison © STEP2 Approach strategically Make Quantity B look more like Quantity A by rewriting a, the area of the circle, in terms of the diameter, d. The area of any circle equals nr, where ris the radius. Because the radius is half the diameter, we can substitute 4for rin the area formula to get ‘nd? [2m quantity 8. simplying, we get Because both quantities contain 7, we could compare ¥ to ©. But let's take it one step further. You know that d is a distance and must be a positive number. That makes it possible to divide both quantities, $ and {, by d and then just compare } to 1. This makes it easy to see that Quantity A is always greater because ; > }.choice (A) is correct. KAPLAN’S ADDITIONAL TIPS FOR QUANTITATIVE COMPARISON QUESTIONS ‘Memorize the Answer Choices. While working through the chapter on Quantitative Comparison, it is a good idea to memorize what the answer choices mean. This is not as difficult as it sounds. Although. the practice questions in this book are shown with the choices for each question, the choices are always the same. The wording and the order never vary. As you work the practice problems, the choices will become second nature to you, and you will get Used to reacting to the questions without reading the four answer choices, thus sav- ing you lots of time. Try to Demonstrate Two Different Relationships Between Quantities. Here's why demonstrating two different relationships between the quantities is an important strategy: If you can demonstrate two different relationships, then choice (D) is correct. There is no need to examine the question further. But how can this be demonstration be done efficiently? A good suggestion is to look at the expression(s) containing a variable and notice the possible values of the variable given the mathematical operation involved. For example, if x can be any real number, and you need to compare (x + 1)? to (x + 1), pick a value for x that will make (x + 1) a fraction between 0 and 1, and then pick a value for x that will make (x + 1) greater than 1. By choosing values for x in this way, you are basing your number choices on mathematical properties you already know: a positive fraction less than one becomes smaller when squared, but a number greater than one grows larger when squared. Latmati: Hose 211 Block A (Opp Dhanmondi Govt Boys) #01921-080848| ‘Banani: Hove 121,Road 4 lock A #01677-481973) Khulna: Rak Properties, House 11,Sensdanga Main Rd #01940-442162| www.grecenternet e = 01120-557481 * grecenter@gmail.com 225 Part Three: Quantitative Reasoning 226 | chaptertt Compare Quantities Piece by Piece. Compare the value of each “piece” in each quantity. If every “piece” in one quantity is greater than a corresponding “piece” in the other quantity, and the operation involved is either addition or multiplication, then the quantity with the greater individual values will have the greater total value Make One Quantity Look Like the Other. When the Quantities A and B are expressed differently, you can often make the com- parison easier by changing the format of one quantity so that It looks like the other. This is a great approach when the quantities look so different that you can't compare them directly. Do the Same Thing to Both Quantities. If the quantities you are given seem too complex to compare immediately, look closely to see if there is an addition, subtraction, multiplication, or division you can perform on both quantities to make them simpler—provided you do not multiply or divide by zero or a negative number. For example, suppose you have the task of comparing w 1 1+ HG 101+ 7 tp where wis greater than 0. To getto the heartofthecomparson, w subtract 1 from both quantities and you have,” compared to 1. To simplify even further, multiply both quantities by (1 + w) and then you can compare w to 1; much simpler. Don’t Be Tricked by Misleading Information. To avoid Quantitative Comparison traps, stay alert and don’t assume anything. If you are using a diagram to answer a question, use only information that is given or infor- mation that you know must be true based on properties or theorems, For instance, don't assume angles are equal or lines are parallel unless it is stated or can be deduced from other information given. ‘A common mistake is to assume that variables represent only positive integers. As you saw when using the Pick Numbers strategy, fractions or negative numbers often show a different relationship between the quantities. Don’t Forget to Consider Other Possibilities. fan answer looks obvious, it may very well be a trap. Consider this situation: a question requires you to think of two integers whose product is 6. Ifyou jump to the conclusion that 2 and 3 are the integers, you will miss several other possibilities. Not only are 1 and 6 possibilities, but there are also pairs of negative integers to consider: ~2 and -3, -1and -6! Lalmati: Hose 211 Block A(Opp Dhanmondi Govt Boye! 401921000848 | ‘Banani: House 121,Road 4, Block A #01677-481978) ‘ulna: Rak Properties, House 11,Senadanga Main Rd #01940-442163| www.grecenter.net = rare Part Three: Quantitative Reasoning ‘Quantitative Comparison Don’t Fall for Look-alikes. Even if two expressions look similar, they may be mathematically different. Be espe- cially careful with expressions involving parentheses or radicals. if you were asked to compare \5x + \5x to Tx, you would not want to fall into the trap of saying the two expressions were equal. Although time is an important factor in taking the GRE, don’t rush to the extent that you do not apply your skills correctly. In this case, \5x + Sx = 2\5x, which is not the same as \TOx unless x = 0. wngrecentarnet =o _aLactane yp aad Cn oan m OT20-557981 Ranier ser 0-5; Okfen Demeaat ene hea anennascial grecenter@gmail.com 227 Part Three: Quantitative Reasoning 228 | Caner QUANTITATIVE COMPARISON PRACTICE SET Try the following Quantitative Comparison questions using the Kaplan Method for ‘Quantitative Comparison. If you're up to the challenge, time yourself; on test day, you'll want to spend only 1.5 minutes on each question. Quantity A. Quantity B 1 et 2x-2 eH2x-1 D Quantity A is greater. Quantity B is greater. © The two quantities are equal. ® The relationship cannot be determined from the information given. x = 2y; y is a positive number. uantity A uantity B 2. ” z ® Quantity A is greater. Quantity B is greater. © The two quantities are equal. ® The relationship cannot be determined from the information given. 4 rand s are positive numbers; grs > 12. Quantity A. Quantity B " 3 . 5 s Quantity A is greater. Quantity B is greater. The two quantities are equal 8668 The relationship cannot be determined from the information given. wrwigrecenternet {sateen 27 Sec Ap Dhaancedl Govt Boy) OI com] =omssua ‘Gre. annitewe fond 4 lek A867? 4978 grecenter@gmail.com ‘Khulna: Rakb Properties, Houce 11, Sonsdangas Main Rd #01940-442162| Part Thre: Quantitative Reasoning ‘Quantitative Comparison | 229 In triangle XYZ, the measure of angle X equals the measure of angle Y. 4 The degree measure of angle Z The degree measure of angle X plus the degree measure of angle ¥ ® Quantity A is greater. @® Quantity B is greater. © The two quantities are equal. D The relationship cannot be determined from the information given. w, med “P, @ ] z ros R square A squareB Quantity A Quantity B x Perimeter of square A Length of WY : Perimeter of square B Length of PR ®D Quantity A is greater. ® Quantity B is greater. D The two quantities are equal. ® The relationship cannot be determined from the information given. ‘www.grecenter.net Lalmatia Hose 2, Bleck (Opp Dhanmand Govt Boy 901921-080648| =omossms Gre. (#) Senanitowe 1, fond lok A #01677 48978 ulna Rai Proprio House 11,Sonadangs Man Re 01940447162 grecenter@gmail.com 230 Part Three: Quantitative Reasoning Chapter 11 QUANTITATIVE COMPARISON PRACTICE SET ANSWERS AND EXPLANATIONS 1B Comparing the two quantities piece by piece, you find that the only difference is the third piece: -2 in Quantity A and —1 in Quantity B. You don’t know the value of x, but whatever it is, x? in Quantity A must have the same value as x? in Quantity B, and 2x in Quantity ‘A must have the same value as 2x in Quantity B. Because any quantity minus 2 must be less than that quantity minus 1, Quantity B is greater than Quantity A. The answer is (B) 28 Replacing the exponent x in Quantity B with the equivalent value given in the centered information, you're comparing 4 with 2. Because y is a positive integer, raising 4 to the exponent 2y will result in a greater value than raising 2 to the exponent 2y. The correct choice is (A). 3. D Do the same thing to both quantities to make them look like the centered information. When you multiply both quantities by 5s, you get qrs in Quantity A and 15 in Quantity B. Because qrs could be any integer greater than 12, ars could be greater than, equal to, or less than 15. Choice (D) is correct. 4D Because angle X = angle ¥, at least two sides of the triangle are equal. You can draw two diagrams with X and ¥ as the base angles of a triangle. In one diagram, make the triangle tall and narrow, so that angle X and angle ¥ are very large and angle Z is very small. In this case, Quantity B is greater. In the second diagram, make the triangle short and wide, so that angle Z is much larger than angle X and angle ¥. In this case, Quantity A is greater. Because more than one relationship between the quantities is possible, the correct answer is (D). www.grecenter.net = 01720-557481 grecenter@gmail.com e Lalmati: Hose 211 Block A (Opp Dhanmondi Govt Boye) #01921-080848| ‘Banani House 121,Road 4 lock A 01677-481978| ‘Khulna: Ral Properties, Hours 11,Senadanga Main Rd #01940-442162| Part Three: Quantitative Reasoning ‘Quantitative Comparison 5c You don't know the exact relationship between square A and square 8, but it doesn't matter. The problem is actually just comparing the ratios of corresponding parts of two squares. The relationship between the specific side lengths of both squares will also exist. between them for any other corresponding length. If a side of one square is twice the length of a side of the second square, the diagonal will also be twice as long. The ratio of the perimeters of the two squares is the same as the ratio of the diagonals. Therefore, the quantities are equal. Choice (C) is correct. {Lalmatia: Hosue 21 Block A (Opp Dhanmondl Govt Boys) #01971-080648| Wanwgrecantarnet ‘= 01720-557481 baakics ted tert ser = 011205 GFE. (# easnatceniin nanauanniascial grecenter@gmail.com 231 Revised GRE’ acy Pag 32-2 AB GRE Center's ye se oo Made Easy For Bangladeshi Graduates Revised GRE -43 >f% etefoa ast 42 ETS Revised GRE Official Guide Barron's New GRE Kaplan New GRE McGraw Hills New GRE Princeton Review New GRE And other New GRE Books eee ecaeae ez Lanai an oa Gpedhnmena coder aon os =: 01720-57481 “af, ©) Fame) jaca sinit oeenniieuss 121 Road Block A #01677-481978| SAT_) hula: Rakibs Prog aoe sere Sonadanga Main Rd #01940-442162 grecenter@gmail.com Bangladesh's cheapest Credit Card and Online Payment Service Provider. @rRe CoE CES Card Tae. eT VISA [BankotAmerica Registration of GRE/SAT/GMAT "Rit C1CT GAIT © Sending Additional Scores FG SIAC OT Paying University & other fees online "WAIT! maa) Paying fees with US Bank Check (Cheque) isa GRECenter's in GRE Center's An incredible ueA |@ collection of "Leaked | @ GRE (2) USA, (3) Math 2 Scrieursta Made Easy Bangladeshi“ Collection (ee ‘verte? GRE “re tton weT wet ore GRE Preparation book | 30 Revised GRE it ‘Rett RCH SRT Core ROH TOT ATT | ‘Everything that you need to know, from early stage like finishing. ‘graduation to taking GRE, TOEFL, Contacting universities, obtaining F-20, facing Embassy 0 first few months in USA! @ Now GRE Center is powered with Online Tutoring & Exam System (OTES) : reared Cre I y's eta Cee ilig ‘OTT TNH GRE Test Taker OFF Vcr Sea CON Fat CS ocePALOOS HUME GRE “AP GR US University admission >Re 74 OCs FRI | SATA CAB: OGRE Download Library OFree assessing Analytical writing OmTPafSs FRIH GRE-TOEFL 991 crawl ester aoe! Onreioe ==8| visa interview face FHT CAR ATE DOR GABE OFFS ae ORE USA CO ORTH Rete U8 OUSA'T AO 0 RoR CCE desired program select FE AM | GOES cae HRI er Fa TE, ‘Sea CRIT USA CO MS/PHD CO ROTTS TOTO SOTO | eC a GRE OU aR www.grecenter.net www.grecenter.net e 01720-557481 grecenter@gmail.com nd Gov Boye) 401921-00004 |: Main fanga Main Ra #01940-482162| Problem Solving INTRODUCTION TO PROBLEM SOLVING Problem Solving can be broken up into several general mathematics categories: alge- bra, arithmetic, number properties, and geometry. In a Problem Solving question, you may be asked to solve a pure math problem or a word problem involving a real-world situation. You will be asked to enter your answer into an onscreen box, select one answer, or select one or more options that correctly answer the problem. The directions for a Problem Solving question requiring a single answer will look like this Directions: Click to select your choice. A Problem Solving question requiring you to select a single answer will look like this. pee hee ES ie Sante avestion A health club charges $35 per month plus $2.50 foreach aerobics class attended. How many aerobics clases were attended ina certain ‘month ifthe total monthly charge was $52.50? 7 8 9 {Lalmati: Hosue 211 Block A(Opp Dhanmondi Govt Boye 401921000848 | ‘Banani: House 121,Road 4, Block A 01677-48197) ‘ulna: Rakib Properties, House 11,Senadanga Main Rd #01940-447163| www.grecenter.net =omossaa Ore grecenter@gmail.com | 233 Part Three: Quantitative Reasoning 234 | chapter12 ‘The directions for a Problem Sol answers will look like this: 1g question requiring you to select one or more Directions: Click to select your choice. Ifa Problem Solving question asks you to select an exact number of choices, you must select that exact number of correct choices for the question to be counted as correct. Otherwise, you must select all the correct choices for the question to be counted as correct. ‘A Problem Solving question requiring you to select one or more answers will Iook like this. [Seas as ee Sule Question If << 1, which ofthe following must be true? Choose all possible answers. aex | wet | 2x>1 wee <1 The directions for a Problem Solving question requiring you to make a Numeric Entry will look like this: Directions: Click in the box and type your numeric answer. Backspace to erase. Enter your answer as an integer or decimal if there is one box or as a fraction if there are two boxes, To enter an integer or decimal, type directly in the box or use the Transfer Display button on the calculator. + Use the backspace key to erase. + Use a hyphen to enter a negative sign; type a hyphen a second time to remove it. The digits will remain. www.grecenter.net Lalmatia: Hosue 2/1, Block A (Opp Dhanmondi Govt Boys’) #01921-080848 | ‘= 01720-557481 ore, Sonanitouse 11,Road Black A 20167-41378 arecenter@gmail.com Khulna: Rakib Properties Mouse 11,Sonadanga Main Rd #01940-442162 Part Three: Quantitative Reasoning Problem Solving 235 + Use a period for a decimal point. + The Transfer Display button will enter your answer directly from the calculator. + Equivalent forms of decimals are all correct. (Example: 0.14 = 0.140) + Enter the exact answer unless the question asks you to round your answer. To enter a fraction, type the numerator and denominator in the appropriate boxes. + Use a hyphen to enter a negative sign. + The Transfer Display button does not work for fractions. + Equivalent forms of fractions are all correct. (Exampl large, reduce fractions to fit in boxes. 3) If number: = 3) If numbers are “45 ‘A Problem Solving question with Numeric Entry will look like this. = = a |comteren a sen cat “The health club charges $35 per month plus $2.50 for each aerobics class attended, How many aerobics classes were attended ina certain month ifthe total ‘monthly charge was $52.50? lasses inborn et Benga ee | worwigrecenternet = a ea ee ee a ae =omo-sme GLE. Sunt tue 1 ted eck cote? 1578 Gearepenoon naeila tera ei Sousa main ntoe sea Part Three: Quantitative Reasoning 236 | chapter12 THE KAPLAN METHOD FOR PROBLEM SOLVING STEP1 Analyze the question. ‘STEP 2 Identify the task. STEP3 Approach strategically. STEP 4 Confirm your answer. How THE KAPLAN METHOD FOR PROBLEM SOLVING WORKS Now let's discuss how the Kaplan Method for Problem Solving works: © STEP1 Analyze the question. Look at what the question is asking, and what area of math is being tested. Also note any particular trends in the answer choices: numbers v. variables, integers v. non- integers, and what information is being given. Unpack as much information as possible. © STEP2 Identify the task. Determine what question is being asked before solving the problem. Ask yourself, “What does the correct answer represent?” The GRE intentionally provides wrong answers for test takers who get the right answer to the wrong question. © STEP3 Approach strategically. Depending on the type of problem, you may use straightforward math—the textbook approach—to calculate your answer, or you may choose one of the following strategies: Picking Numbers, Backsolving, or Strategic Guessing. When Picking Numbers to substitute for variables, chose numbers that are manageable and fit the description given in the problem. Backsolving is another form of Picking Numbers, where you'll start with one of the answer choices, and plug that choice back into the question. Lastly, Strategic Guessing can be a great time-saver on the GRE— being able to make a smart guess on a question is preferable to taking too much time and thus compromising your time and ability to answer other questions correctly. ‘www.grecenter.net {matin Honue 2, Block (Opp Dhanmondi ov Boyt #0192 00048 = 01720-557481 ore, SananiHowe 12, Row lock As01677-481573 greventeriiymeiLcom ulna Rak Properties, House 11, Sonsdange Main Ré#01940-442162| Part Three: Quantitative Reasoning Problem Solving © STEP 4 Confirm your answer. ‘Check that your answer makes sense. Also check that you answered the question that was asked. How To APPLY THE KAPLAN METHOD FOR PROBLEM SOLVING Now let's apply the Kaplan Method to a Problem Solving question: In a bag of candy, 7 of the candies are cherry-flavored, 8 are lemon, and 5 are grape. If a candy is chosen randomly from the bag, what is the probability that, the candy is not lemon? © STEP Analyze the question. You are given the number of candies in a bag, and asked to identify the probability that a randomly selected candy is not lemon-flavored. You will have to type your answer into the box. © STEP2 Identify the task. i Number of Favorable Outcomes The probability of an eventis defined as “Numer of Favorable Outcomes. you will need to find the number of favorable outcomes (those in which you don't choose a lemon candy) and the total number of possible outcomes. © STEP 3 Approach strategically. There are 20 candies in the bag, so there are 20 possible outcomes, Of all the can- dies, 12 are not lemon, so there are 12 favorable outcomes. So, the probability of not lemon is 32. You should avoid reducing fractions for Numeric Entry questions, since all equivalent forms will be counted as correct. Save your time to use elsewhere, and limit your risk of committing an error in calculation. wwwgrecenternet {atone 27 Sec AOpp haan Gort Boye) 01 cm 2: 01720-55481 ore. annitee 1, ond lek A067? 43978 ee tte ll rept Hee datas ed presen 237 Part Three: Quantitative Reasoning 238 | chapteriz © STEPS Confirm your answer. Although it might be fun to get a bag of candies and check your answer in a real-world way, it's not practical, especially on test day. A more practical check would be to find the probability of choosing a lemon candy at random to be certain that: P\lemon) = 1 — Pinot lemon). There are 8 lemion candies out of 20, so this check can be done easily. P\lemon) 2 1 ~ P{not lemon) 824-12 Bis) 20 82,202 20 * 20 20 &-8 20 ~ 20 This check is a way to confirm that the correct numbers have been used in the problem and the correct answer has been found. Now let’s apply the Kaplan Method to a second Problem Solving question: When nis divided by 14, the remainder is 10. What is the remainder when n is divided by 72 086908 © STEP ‘Analyze the question. In this question, you are asked to compare the relationship between the numbers 14 and 7 used as divisors. Be careful; you may be thinking of choosing 14 = 7 = 2 or 10 + 2 = 5. But these are both trap answer choices, because the question also involves using a remainder. © STEP2 Identify the task. The task is to use the fact that division of a number, n, by 14 yields a remainder of 10 to identify the remainder when the same number is divided by 7. wurgrecenternet =o {anton 27 SecA Opp haancndl Got Boy) OI cm =omosoms OLE, annktee 12, ond lek A067? 48978 grecenter@gmail.com Khulna: Rak Properties, House 11,Sonadanga Main Rd #01990-442162| Part Three: Quantitative Reasoning. Problem Solving © STEP 3 Approach strategically. A good strategy for this question is to pick a number for n that satisfies the condition for division by 14 and then see what happens when it is divided by 7. ‘Any number divided by itself will give a remainder of zero. So if we need a remainder of 10, we want a number that is 10 more than the number we are dividing by. 24 is a great number to pick here because when we try 24: 24 = 14 = 1 Remainder 10 Now that we've confirmed that 24 works, we answer the question that's being asked. Divide 24 by 7: 24+ 3 Remainder 3 Answer Choice (B) is the correct answer. ‘® STEP 4 Confirm your answer. You can quickly double-check your work, or try another number for n that results ina remainder of 10 when divided by 14: 38 + 14 = 2 Remainder 10, and 38 + 7 = 5 Remainder 3 So the remainder is 3 in each case. The correct answer is (B). www.grecenter.net Lalmatia: Hosue 2/1, Block A (Opp Dhanmondi Govt Boys’) #01921-080848| = 01720-557481 gre (S) Sanan Howe 171, oud 4 Block A 01677481978 nina: Properties Hse 1, Senadage sn he 90-4422 grecenter@gmail.com 239 Part Three: Quantitative Reasoning 240 | chapter12 Now let’s apply the Kaplan Method to a third Problem Solving question: ‘The line 4x + 6y = 24 passes through which of the following points? Indicate all possible answers. Ao B 23) @2) B64 O @-) © STEP1 Analyze the question. This question is about a line on the coordinate plane. The equation is a function that represents a line. The numbers in the parentheses represent points (x, y) that are men- tioned in the equation. © STEP2 Identify the task. Your job is to identify which of the given points lie on the line. A line passes through 2 point if the coordinates of the point make the equation of the line true, so this is the same as saying that you need to find out which points, when plugged into the equa- tion, make the equation true. © STEP 3 Approach strategically. You need to find all correct answers, so test all of them. Substitute the first coordinate for x and the second coordinate for y. (A) Test (0, 4): &x + 6y = 24 + 4(0) + 6(4) = 0 + 24 = 24. This works. (B) Test (2, 3): 4x + 6y = 24> 4(2) + 6(3) = 8 + 18 # 24. Eliminate, (C) Test (3, 2): 4x + 6y = 24 > 4(3) + 6(2) = 24, This works. 2+ 12 {D) Test (5, 4): 4x + 6y = 24 > 4(5) + 6(4) = 20 + 24 24. Eliminate. (E) Test (9, -1): 4x + 6y = 24 + 4(9) + 6(—1) = 36 — 6 24. Eliminate. So, answer choices (A) and (C) are correct. wwwagrecenternet {ante on 27 ec AOppaancndl Govt Boy) O19 cw 2: 01720-55481 ore. Tannen 1, ond lek #6774978 grecenter@gmail.com Khulna Rak Properties, House 11,Sonsdangs Main Ré#01940-442162| Part Three: Quantitative Reasoning Problem Solving STEP 4 Confirm your answer. Double-check your work to make sure you haven't made any careless errors, such as mistakenly plugging in a value for x when dealing with the variable y. KAPLAN’S ADDITIONAL TIPS FOR PROBLEM SOLVING Choose an Efficient Strategy The GRE is not a traditional math test that requires that you show your work before you get credit, testing the process as well as the answer. The GRE tests only the answer— not how you found it. Because time is often your biggest concern on the GRE, the best way to each solution is often the quickest way, and the quickest way is often not straightforward math, Through practice, you'll become familiar with approaching each question in a more strategic way. Rely on Kaplan Math Strategies Kaplan strategies are a way to use reasoning in conjunction with mathematics to answer a question quickly. There may also be cases in which you can combine approaches: for example, using straightforward math to simplify an equation, then picking manageable numbers for the variables to solve that equation. PICKING NUMBERS Problems that seem difficult are good candidates for the Picking Numbers strategy. ‘They include problems where either the question or the answer choices have variables; the problem tests a number property you don't recall; or the problem and the answer choices deal with percents or fractions, without using actual values. BACKSOLVING Backsolving is a similar strategy to Picking Numbers, except that you'll use one of the answer choices as the number to pick. Remember, numerical answer choices are always in ascending or descending order. Use that information to your advantage when Using Backsolving. Start with either (B) or (0) first, because you'll have a 40% chance that you can find the correct answer based on your first round of calculations. Then reason whether the choice you started with is correct, too large, or too small. STRATEGIC GUESSING This is a good strategy if you can eliminate choices by applying number-property rules or by estimating because gaps between answer choices are wide. If some of the choices are out of the realm of possibility, eliminate them and move on. www.grecenter.net 2 r gre _omosmnet, Ol tren Lalmati:Hoowe 2/1, lock A(Opp Dhanmondi Govt Boy 401921-080848 ‘Banani: House 121, Road 6, Block A.#01677-481978| Kua: Rak Properties, Mouse 11,Senadanga Main Rd #01940-442162 art Three: Quantitative Reasoning 20a | Gone PROBLEM SOLVING PRACTICE SET Try the following Problem Solving questions using the Kaplan Method for Problem Solving. If you're up to the challenge, time yourself; on test day, you'll want to spend only about 2 minutes on each question. 1. Ifr = 35,5 = 5t,t = 2u, and u * 0, what is the value of 2 @D 30 ®D 0 @ 150 ®D 300 ® 600 2. In the diagram, |, is parallel to J,, The measure of angle q is 40 degrees. What is the sum of the measures of the acute angles shown in the diagram? degrees 3. At Central Park Zoo, the ratio of sea lions to penguins is 4:11. If there are 84 more penguins than sea lions, how many sea lions are there? @D u @®D % © 8 Dr @ 1 www.grecenter.net 080848 e {ata Hore 2B App harmon Got Boys) 0 = 01720-557481 Ore. (=) ‘Banani Howe 121,Road Block A#01677-481970 grecenter@gmail.com ina Raib Properties Howe 1 Senedange Main hd 40140-87162 Part Three: Quantitative Reasoning. Problem Solving 203 4. Which of the following are prime numbers between. Sand ee Indicate all possible answers 5. The figure above is made up of 3 squares having the same side length. If the perimeter of the figure is 40 units, what is the area in square units? @® so — 9 www.grecenter.net {Latmatia: Hosue 21 Blok A (Opp Dhanmondl Govt Boys) #01921-080848| ore. orton 1, fond lo 10167740370 poemeenion ulna: Properins Hse 1, Senange iin Re e192 = 01720-557481 244 Part Three: Quantitative Reasoning Chapter 12 PROBLEM SOLVING PRACTICE SET ANSWERS AND EXPLANATIONS 1 E The other variables all build upon u, so use the Picking Numbers strategy: pick a small number for u, and find the values for rs and t. For instance, if u = 1, then ¢ = 2u, sot = 2; 5 = St, $05 = 10; andr = 35, sor = 30. pst _ 30X10 x2 we TXTXT So, 00, The correct answer is (E). 2. 160 In the diagram, there are four acute angles and four obtuse angles created when the parallel lines are cut by the transversal. if angle q has a measure of 40", then angles s, fh, and falso have a measure of 40°. Therefore, the sum of their degree measures is 160. 3¢ You need to find the number of sea lions, and there are fewer sea lions than penguins, so starting small is a good idea. You can use the Backsolving strategy; start with choice (B), 36. If there are 36 sea lions, then there are 36 + 84 = 120 penguins, and the ratio of sea lions to penguins is & = J This ratios less than +4, so your answer must be larger. If we try (D), there are 72 sea lions and there are 72 + 84 = 156 penguins, and the ratio of sea fons to penguinsis 22 = since this aio is too lrg, the correct answer must be () 4 ACD You need to find a range of values between two improper fractions. First, change the improper fractions to mixed numbers: 3 2}and 3 positive integer with only two distinct factors, 1 and itself. he prime numbers in the answer 2. Now, a prime number is a choices are 3, 5, and 7, and they are all between 24 and 83. So the correct answers are (A), (Q), and (0). 5. B There are 8 side lengths of the squares that make up the perimeter, which you are told is 40, So, each side of each square must be 5 units. The area of each square can found by squaring one side, so each square has an area of 25 square units. Since there are three squares, the total area of the figure is 75 square units. The correct answer is (B) www.grecenternet = 01120-557481 grecenter@gmail.com ie e Latmati: Hose 211 Block A (Opp Dhanmondi Govt Boys) #01921-080848| ‘Banani: Houee 121,Road Block A #01677-481973) ‘Khulna: Rak Properties, House 11,Sensdanga Main Rd #01940-442162| GRE Center: Country's only complete GRE, GMAT & SAT provider ‘ieee GRE (ETS) wi cee ‘weet GRE/ TOEFL cere BORD See OFT AER ICY, PACT, eaten om oer aa Tet score card AQT BE Of SINICA ree faa | ETS or are appointment # Sst SIs Texas often arti GRE/ TOEFL score card receive FAS, WIRE OT ST HE TOE WG SIO GT aABHICT Ce 4 a0 OFF aR a Visit our website ‘www grecenter.net where you can participate Free GRE Analytical Writing evaluation. Our teachers will grade your writing. Bea proud student of an institute that guides all other coaching centers of the country. aq OAs FAT COG Slewat? GRE 3 1 ors sem specialized aftr: GRE, GMAT, SAT, USMLE - eaten exore course USA Gir! USA ate ono af were gear aT | EGER 48 fs = GRE/GMAT/SATS FO fogs Arora Ag Fal AA | A aE Free TOEFL | Cre Ae CHAIR FCT cle VEY, teaching qualification, interpersonal communication skill ae ane cere eks GM acre program <7 faculty Feat Fret | seed Gteat Gino GM materials =f GTA | Research <4 WHR develop #4 TAKE ABE Rs | MMR afd Fem Lecture sheet oo UR 447 ace oR By | CED xt eve 343) ser OS, aH ET aT aS A feo ea aT | CATaT ACT COG CARAT OAT “ASE” am ATTA TOAD oer Fen atc | rea weer BF 4B roe FF CROAT TR 4 ET TT HICH AT GE 2 er vocabulary he, oat ore ct 16 F146 GR Rus 9H OE GIA afObIC AZ| Pray orb CIT Ta ICT FCW OTE AT ARE GET HE PA Trea Bre HAD IS TH CE CT FR BS TG CA Lom Powder He HATHA Fa TH COTA FT Har RTO rR MK HO OT | Gas mer CD we Pras ete ‘abot @ E-book & crew wr | AOA F eee | aE ARS Rt | [WeOH Ft miss WA Ton FIO AT CHA Toa CR Fat TH CE DRT TO As SF Rat ea oT repeat P00 *147 1 Cie aR Remand Refte cicra stots orgs | www.grecenternet rare ss 01720-557481 grecenter@gmail.com almati:Hosue 21 Block A(Opp Dhanmonel Govt Boys) #0192-080848| nanicHouse 121, Road 4 Block A.#01677-481978 House 11,Sonadanga Main Ra #01940.042162| | | UC ats ee Data Interpretation INTRODUCTION TO DATA INTERPRETATION QUESTIONS Data Interpretation questions are based on information located in tables or graphs and are often statistics-oriented. The data may be located in one table or graph, but you might also need to extract data from two or more tables or graphs. There will be 2 set of questions for you to answer based on each data presentation. You may be asked to choose one or more answers from a set of answer choices or to enter your answer in a Numeric Entry field The directions for Data Interpretation questions will look like this: Questions 1-5 are based on the following table. PERCENT OF SALES PER CLIENT FOR CURTAIN FABRIC OVER THREE MONTHS. May The Home Touch 45% Curtains Unlimited 30% Max's Curtain Supply 9% Valances by Val 13% Wendy's Windows 3% June uly 25% 23% 23% 20% 9% 48% 3396 22% 10% 7% www.grecenter.net = 01720-557481 ‘Lalmtia:Hosue 21, Block A (Opp Dhanmondi Govt Boys) #01921-080848| Banani: House 121, oad 4, Block A.#01677-481978| eae sat Btn ($F) anna 245 Part Three: Quantitative Reasoning 246 | Chapters3 A Data Interpretation question that requires you to choose exactly one correct answer will look like this: ome SEs samo [If total sales for curtain fabric in July were $150,000, thaw much revenue did The Home Touch account fort A Data Interpretation question that requit apply will look like this: s you to select all the answer choices that sero comton as | Samet. In the months of June and July, which clients accounted for more than 15% of sales each month? Choose all that apply. Curtains Unlimited Max’ Curtain Supply ‘Valances by Val ‘Wendy's Windows "The Home Touch etna ‘www.grecenter.net Lalmati: Hoave 271 Block A (Opp Dhanmondi Govt Boys!) 401921-080848| =: 01720-557481 Olen Santee hand lok A 416774378 grecenter@gmailcom ‘ulna: Rakib Properties, House 11,Senadanga Main Rd #01940-447163| Part Three: Quantitative Reasoning Datalnterpretation | 247 A Data Interpretation question that requires you to enter your numeric answer in a box will look like this: ae Eyre In May, the two clients representing the greatest percentages of sales accounted for $81,000 in sles. What were the total sales forthe month of May? [ecient Ctainentox sayy none tag enc, ‘www.grecenter.net = 01720-557481 grecenter@gmail.com {Llmatia:Hosue 21, Block A (Opp Dhanmondi Govt Boys) 401921-080848 Khulna: Rab Properties, our 1,Sonadanga Min Rd #01940-447162 Banani: House 121, oad 4, Block A.#01677-481978| Part Three: Quantitative Reasoning 208 | Chapters THE KAPLAN METHOD FOR DATA INTERPRETATION STEP 1 Analyze the tables and graphs. STEP 2 Approach strategically. How THE KAPLAN METHOD FOR DATA INTERPRETATION WORKS Now let’s discuss how the Kaplan Method for Data Interpretation works. © STEP 1 Analyze the tables and graphs. Tables, graphs, and charts often come in pairs that are linked in some way (for example: a manufacturer's total revenue and its revenue by product line). Familiarize yourself with the information in both graphs (or tables) and with how the two are related before attacking the questions. Scan the figures for these components: + Title. Read the charts’ titles to ensure you can get to the right chart or graph quickly. + Scale, Check the units of measurement. Does the graph measure miles per minute or hour? Missing the units can drastically change your answer. + Notes. Read any accompanying notes—the GRE will typically give you information only if itis helpful or even critical to getting the correct answer. + Key. If there are multiple bars or lines on a graph, make sure you understand the key so you can match up the correct quantities with the correct items. © STEP2 Approach strategically. Data Interpretation questions are designed to test your understanding of fractions and percents and your attention to detall. Taking a split second to make sure you answered the right question can make the difference between a correct answer and a question answered wrong on a technicality. Questions tend to become more complex as you move through a set. For instance, if a question set contains two graphs, the first question likely refers to just one graph. A later question will most often combine data from both graphs. if you don’t use both ‘graphs for this later question, the chances are good you have missed something. et Lalmatia: Hosue 2/1, Block A (Opp Dhanmondi Govt Boys’) #01921-080848 | ‘=: 01720-557481 gre Paral Howne 3, fond lock A016 43373 recenter@gmailicom ‘ulna: Rak Properties, House 11,Senadanga Main Rd #01940-442163| Part Three: Quantitative Reasoning Data interpretation | 249 No matter how difficult graph questions appear at first glance, you can usually simplify single-answer multiple-choice questions by taking advantage of their answer-choice format. By approximating the answer rather than calculating it wherever possible, you can quickly identify the right one. As we saw with Problem Solving, estimation is one of the fastest ways to solve math problems. Data Interpretation questions benefit from this strategy, as they tend to be the most time-consuming questions to answer. How To ApPLy THE KAPLAN METHOD FOR DATA INTERPRETATION Now let’s apply the Kaplan Method to a Data Interpretation question: (Climograph of City 20 80 18 70 0 CEL EK ES CAS So “ae Mean daily maximum temperature CC) © Mean daly ninimum temperature (°C) {B_ Mean toa rainfall (mm) ‘The Tourism Board of City S uses the information provided in the climograph to market the city as a tourist destination. One criterion is that the average monthly rainfall be less than 60 millimeters. What fraction of the months meets this criterion? irra —S wwwgrecenternet ag Laat: Hone 2, Block (Opp Dhanmond Gort Boy #0192 o90848 =omosoms OLE, ean owe 1, ond Dock s01677 409978 qecacuenruicon ulna: Ral Proertien House 1, Sonadang Man Re 01340-44216 Part Three: Quantitative Reasoning 250 | Chapters © STEP1 ‘Analyze the tables and graphs. Take the analysis of the graph step-by-step. Start with the title of the graph to verify that the data given is for City S. Then take note of the scale for each type of information— degrees Celsius for temperature and millimeters for rainfall. There is data for each month of the year, which means you will not have to convert the units to answer the question that's being asked. © STEP2 Approach strategically. The question asks only about rainfall; that data is given by the bars on the graph. ‘According to the bars, rainfall is greater than 60 mm in Aug, Sep, Oct, and Nov. That’s 4 of 12 months that do not meet the criteria, so 8 of 12 months do meet it. You may enter the fraction directly into the boxes, and your answer will be accepted. It is not required that you reduce it Now let's apply the Kaplan Method to a second Data Interpretation question: Customers Who Switched Service Provider (anmilions of customers) | 2004" 2005" 2006" 2007" 2008 2008 = AwB = BWA Company A Profit 2008 Bemw Bi Phinx Di riny D Punz 409% www.grecenter.net Lalmatia:Hosue 21, Block A (Opp Dhanmondi Govt Boys) #01921-080848| ‘=: 01720-55481 ore. Bananouse 1 fond lc A 01677-41578 rctacaere salle! ane ss rope aT seagate pos Part Three: Quantitative Reasoning Data interpretation In 2008, Company A had a total profit of $220 million. If half of the customers who switched to Company A were responsible for half of the profit for Plan X, how much did those customers contribute per person towards Company A's profit for the year? @® suo ® 813.75 © s20.25 ® 32750 ® $55.00 © STEP 1 Analyze the tables and graphs. This question has information about numbers of customers switching service providers for various years. It also has information about one company's profit for the year 2008, 0 the data in the two graphs will be linked by the year 2008. © STEP2 Approach strategically. ‘Approach the question methodically starting with identifying the number of customers who switched to Company A. It indicates that 4 million customers switched to Company A. This is the only information needed from the top graph, The pie chart shows the breakdown of profit from the various plans offered and indi- cates that 25% of the profit came from Plan X, The other information you need to get to the correct answer is given in the question stem: + Profit of $220 million + Half of the customers who switched were responsible for half of Plan X's profits. Now that your information is organized, all you need to do is the calculation. Plan X accounts for 2596 of $220 million = $55 million. Half of $55 million is $27.5 million. If 4 million people switched, then half of the people who switched would be 2 million. The last step is to divide 27.5 by 2 (you can drop the zeroes in the millions because ‘they will cancel out): $275 + 2 = $13.75. The correct choice is (B). worw.grecenternet ama Hose 2 Bloc (Opp Dhanmondi Govt Boye 01021 Ome =omosmn GLE (S) Banaue 12, fond 4 lock 40167 401978 grecenteréymall.corm ulna: fab Properties Howe 1 Sonedange Main hd 4940-42162 Part Three: Quantitative Reasoning 252 | chapteri3 Now let’s apply the Kaplan Method to a third Data Interpretation question: (Customers Who Switched Service Provider (Gn millions of customers) 2008 2008 2006" 2007" 2008 2009 = ADB = BWA ‘Company A Profit 2008 15% Gi Pinw 40% ie WH Planx DD Piany D Pianz 259% ‘The management of Company B is most interested in the data for the years in which there were at least one million more customers who switched from Company A to Company B than switched from Company B to Company A. In which years did this happen? Choose all that apply. 2005 2006 2007 2008 2009 {Lamatia: Hosue 21, Block A (Opp Ohanmendl Govt Boys) 401971-060648, Banani: House 121, oad 4, Block A.#01677-481978| Khulna: Rakib Properties, ouse 11,Sonadanga Main Rd #01940-447162 ‘www.grecenter.net 01720-557481 grecenter@gmail.com Part Three: Quantitative Reasoning Data Interpretation © STEP 1 Analyze the tables and graphs. This question asks for a comparison of facts between Company A and Company 8. Take time to verify which line in the top graph represents customers switching to Company A and which line represents customers switching to Company B. Confirm that the title states that the data are given in millions and then look at the scale on the line graph © STEP 2 Approach strategically. After examining the line graph carefully, you are ready to gather the information needed to answer the question. The years that satisfy the requirement are those years for which the line representing A to B is at least one full horizontal row above the line representing B to A. Read the graph carefully because you must identify all the correct, choices to get credit for a correct answer. ‘When you are clear what to look for on the graph, start from the left and identify the years 2005 and 2009 as those in which at least one million more customers switched from A to B than switched from B to A. These are choices (A) and (E). KAPLAN’S ADDITIONAL TIPS FOR DATA INTERPRETATION QUESTIONS Stow down There's always a lot going on in Data Interpretation problems—both in the charts and in the questions themselves. if you slow down the first time through, you can avoid calculation errors and having to reread the questions and charts. Pace Yourself Wisely To ensure that you score as many points on the exam as possible, use the allotted time for a section wisely. Remember that each question type has the same value. If you must miss a few questions in a section, make them the ones that would take you the longest to answer, not the ones at the end of the section that you simply didn't get to, but could have answered correctly, Data Interpretation questions are generally some of the more time consuming ones to answer, and if answering them isn’t one of your strong suits, save them for the end. www.grecenter.net 01120-557481 grecenter@gmail.com Geen {Lalmati:Hosue 211 Block A(Opp Dhanmondi Govt Boye 401921000848 | ‘Banani: House 121,Road 4, Block A #01677-481978) ‘ulna: Rak Properties, House 11,Senadanga Main Rd #01940-447163| 253 Part Three: Quantitative Reasoning 254 | Cpr DATA INTERPRETATION PRACTICE SET Try the following Data Interpretation questions using the Kaplan Method for Data Interpretation. If you're up to the challange, time yourself; on test day, you'll want to spend only about 2 minutes on each question. Questions 1-5 are based on the following graphs. MEGACORP INC. REVENUE AND PROFIT DISTRIBUTION FOR FOOD AND NONFOOD RELATED OPERATIONS, 2004-2009 ‘Total Revenues ‘Total Profits 82 | Co Foopretarep Gi Nonroop retaTeD ‘Note: Drawn to Seale PERCENT OF REVENUES FROM FOOD-RELATED. OPERATIONS IN 2009 BY CATEGORY Frozen Foods = Boy pene \ a / Soft Drinks / 13% SnackFoods \ www.grecenter.net e {Lalmatia: Hose 2/1, Block A (Opp Dhanmondi Govt Boys) #01921-080848 somos OLE ‘anaiowe 121, oud 4 Block A w01677 401978 grecenter@ymail.com ulna Rak Properties, House 11, Sonadanga Main Rd #01940-442162| Part Three: Quantitative Reasoning Datalnterpretation | 255 Approximately how much did total revenues increase from 2004 to 2007? ® $0.5 billion $1.5 billion © $4 billion ® $45 billion ® ss dillion For the year in which profits from food-related operations increased over the previous year, total revenues were approximately ® $35 billion. $4.5 billion, © $57 billion. ® $6 billion @ $8 billion. In 2008, total profits represented approximately what percent of Megacorp’s total revenues? D 50% 20% D 10% D % eS |. For the first year in which revenues from nonfood-related operations surpassed $4.5 billion, total profits were approximately ® $250 million. ® $450 million. © $550 million D $650 million. ® $800 million. In 2009, how many millions of dollars were revenues from frozen food operations? millions of dollars www.grecenter.net 01720-557481 grecenter@gmail.com e {ima Honus 2,80 (Opp Dhanmondi ov Boyt 40197-00048 Gr: (=| Bana Howne 2, Road lock A 01677-40370 natal Proprio Howe 1 Sonadange Man 4040 442162 Part Three: Quantitative Reasoning 256 | Ger DATA INTERPRETATION PRACTICE SET ANSWERS AND EXPLANATIONS 1D This question asks about total revenues, so you should refer to the left bar graph. Each bar in the graph has two components, but you want to look at the total height of the bars for 2004 and 2007 because the question asks about total revenue. Total revenues for 2004 appear to be $3 billion, and for 2007 they appear to be about $75 billion. So the increase is roughly $75 billion — $3 billion = $4.5 billion. Answer choice (D) is correct. 2 You have to refer to both bar graphs to answer this question. First, refer to the right bar graph to find the lone year in which food-related profits increased over the previous year—the only year in which the unshaded portion of the bar increases in size is 2008. Now that you've zeroed in on the year, refer to the left bar graph to determine the total revenues for that year, which appear to be about $8 billion. Answer choice (E) is correct. B.C This is a percent question, so start with the bar graphs. You need the figures from both food-related and nonfood-related sources, so look at the total height of the bars. From the right bar graph, the total profits for 2008 appear to be $800 million; from the left bar graph, total revenues for that year appear to be $8 billion (ie. $8,000 million). Now, convert the part/whole into a percent: 800 million _ 800 million, _ 1 Bbillion ~ 8,000 millon ~ 10 ~ '** 4 First, find the year for which revenues from nonfood-related operations surpassed $4.5 billion on the left bar graph. Finding the correct bar is made more difficult by the fact that you have to deal with the shaded portion, which is at the top of the bar, not at the bottom. Looking carefully, you should then see that 2007 is the year in question. The question asks for total profits, so once again refer to the right bar graph, and you'll see the profits for that year are ‘around $800 million. This matches answer choice (E). 5. 600 Finally, you have a question that refers to the pie chart. You are asked about revenues from frozen food operations, and the pie chart tells you that frozen foods represent 20 percent of all food-related revenues for 2009, To convert this into an amount, you need to locate the ‘amount of food-related revenues for 2009, Once again, refer to the left bar graph, where youll find that food-related revenues in 2009 were $3 billion, or $3,000 million. Then 20 percent of $3,000 million is $600 million. www.grecenter.net _Lalmatia: Hosue 2/1, Block A (Opp Dhanmondi Govt Boys’) #01921-080848 | = 01120-557481 Oren aunt Howe iRond sick 1677-91978 grecenter@gmail.com Kulna:Rak Properties, House 11,Sonadanga Main Ra #01940-447162 GRE Center: Te ek ee sd GRE, GMAT & SAT provider ‘FoR GRE (ETS) a cece weerewet GRE/ TOEFL cesené CS Te OFT AM oR iC, PACT, ween ae | coer aR tort score card "RT ERR CHS UIE sree fit ETS tet appointment a Strat wate Texas wc Mata GRE/ TOEFL score card receive FRG, OS OT MAT IE COT A | TRIOS CHA SABA OE aC OFF aR eT Visit our website ‘www grecenter.net where you can participate Free GRE Analytical Writing evaluation. Our teachers will grade your writing. Be a proud student of an institute that guides all other coaching centers of the country. a Saat WAT COG StewWat? GRE «8 Ba ors eae specialized efebtt| GRE, GMAT, SAT, USMLE - strom exore® course USA ARE! USA cre Rote a were eT OTT | 48 ft'a HR GRE/GMAT/SATS Wr fPgcot ADPers Hg Fal A A | I AGH Free TOEFL | FOS FAS COMIAOR ACT CCS UAC, teaching qualification, interpersonal communication skill aR wine capo feo GME acre program <7 faculty Feat fem @t | sPeid Gttat Gino GM materials yf oot Research a mor develop at WaT ABB wie GIT A Flom Lecture sheet om OR ae soe oA By CED *ete owe (343) seo OD, aH eT Co TES Poo eH at) erat ACE COD crea Oa “AAT” a ATH ast ore Rea ace | CRO oe wes B48 Gee a root wR EN GT FIT AR GEEEEED 2 sere vocabulary “ei, ea eet afd 16 B10 GR ROS APH aT oT aTOBIE AT ‘fea cri CIE Ta SIC FICK OT AT AHA OCT & HE PMN Toa Mee RG HR FP FHT CH GR BS TS CF Lom FET Hor Fe FATHA FAT TH COTA FT A HRT Rt AME HT OH | Goes wer CD ww Pero eater webowts @ E-book & creat wr | aOMAt B eee | ATE afta Bet | ‘WSO FT miss WH We FIO TT OA TOR FCN Fat TE | CHE DRT TO As SF Pat a repeat PHO a1 GiisnaR Qo cumicealcad Rete cers storm org www.grecenter.net e = 01720-557481 ‘( grecenter@gmail.com LalmatisHowve vlna Rak Pro ‘Block A(Opp Dhanmondi Govt Boyt #01921-08064e ni: House 121,Road 4, lock A #01677-48197| House 11,Sonadanga Main Ra 4o1s40.442162 Quantitative Reasoning Practice Set In this section, you will take a practice test composed of 120 questions. This section has been divided into two parts to allow you to check your answers at the halfway mark You will use a diagnostic tool at that point to help you learn from your mistakes and continue with more awareness of the traps you may encounter in the second set. REVIEW OF THE KAPLAN METHOD FOR QUANTITATIVE REASONING QUESTION TYPES Review the steps and strategies you have studied for answering each type of Quantitative Reasoning question quickly, efficiently, and correctly before starting your Practice Sets. THE KAPLAN METHOD FOR QUANTITATIVE COMPARISONS STEP 1 Analyze the centered information and quantities. STEP 2 Approach strategically. LalmatiasHosue 21, Block A(Opp Dhanmondi Govt Boys) #01921-080848 ng | 257 Banani: House 121, Road 4, Block A.#01677-481978| Khulna: Raklb Properties, House 11, Sonadanga Main Rd #01940-447162 ‘www.grecenter.net aomossue Ore. grecenter@gmail.com Part Three: Quantitative Reasoning 258 | chaptersa THE KAPLAN METHOD FOR PROBLEM SOLVING STEP1 Analyze the question. STEP 2 Identify the task. STEP 3 Approach strategically. STEP 4 Confirm your answer. THE KAPLAN METHOD FOR DATA INTERPRETATION STEP 1 Analyze the tables and graphs. ‘STEP 2 Approach strategically. QUANTITATIVE REASONING PRACTICE SET 1 NumBers All numbers are real numbers. FiguRES The position of points, lines, angles, and so on may be assumed to be in the order shown; all lengths and angle measures may be assumed to be positive. Lines shown as straight may be assumed to be straight. Figures lie in the plane of the paper unless otherwise stated. Figures that accompany questions are intended to provide useful information. However, unless a note states that figure has been drawn to scale, you should solve the problems by using your knowledge of mathematics, not by estimation or measurement. DirEcTIONS Each of the following questions, 1-20, consists of two quantities, Quantity A and Quantity B. You are to compare the two quantities and choose ® if Quantity A is greater. if Quantity B is greater. ® if the two quantities are equal. ® if the relationship cannot be determined from the information given. ‘www.grecenter.net Imai: Hosue 2/1, Bleck A(Opp Dhanmondi Govt Boy) #01921-080848 anani: House 121, Road 4, Block A.90167-481878 ulna: Ral Properties, House 11,Senadanga Main Rd #01940-442162| = rare COMMON INFORMATION Part Three: Quantitative Reasoning ‘Quantitative Reasoning Practice Set 259 |n.a question, information concerning one or both of the quantities to be compared is centered above the two quantities. A symbol that appears in both quantities represents the same thing in Quantity A as it does in Quantity B. 1, Quantity A Quantity B The number The number of of ways ‘ways 10 people 6 people can can be arranged be arranged ina line of in aline of 3 people where 3 people where order does not order matters. matter. D Auantity A is greater. Quantity B is greater. © The two quantities are equal. ® The relationship cannot be determined from the information given. 2. QuantityA Quantity B atcte bt+dtf ®D Quantity A is greater. Quantity B is greater. © The two quantities are equal. ® The relationship cannot be determined from the information given, 3, Ipt3=r apt7=s Quantity A Quantity B r s D Quantity A is greater. ®D Quantity B is greater. © The two quantities are equal. ®D The relationship cannot be determined from the information given. The original cost of a shirt is x dollars. Quantity A Quantity B x A 10% increase of x followed by a 10% decrease D Quantity A is greater. Quantity B is greater. © The two quantities are equal. @D The relationship cannot be determined from the information given, Lalmatia: Hosue 21 Block A (Opp Dhanmondl Govt Boys) #01971-080648| ‘Banani: House 121,Road 4 Block A #01677-481878| worwigrecenternet = omosia! OLE, nln: Proparos Hone, Senadnge Nain Re 0190-40716 grecenter@gmail.com Part Three: Quantitative Reasoning 260 | chapters6 ‘There are x dictionaries in a bookstore. After + of them were purchased, 10 more dictionaries ‘were shipped in, bringing the total number of dictionaries to 52. 5. Quantity A Quantity B x 50 ®D Quantity A is greater. Quantity B is greater. ® The two quantities are equal. ® The relationship cannot be determined from the information given. There are m people in a room. One third of them leave the room. Four people enter the room. There are now © of the original number of people in the room. 6. QuantityA Quantity B n 20 ® Quantity A is greater. Quantity B is greater. ‘© The two quantities are equal. ® The relationship cannot be determined from the information given. ‘Two rectangles with dimensions 2 meters by 4 meters overlap to form the figure above. All the angles shown measure 90°. QuantityA Quantity B ‘The perimeter 16 of the figure, in meters ®D Quantity A is greater. D Quantity B is greater. © The two quantities are equal. ® The relationship cannot be determined from the information given. x is an integer. 10>q 13. QuantityA Quantity B peq qep 15. Quantity A Quantity B x+y 180 ® Quantity A is greater. ®D Quantity B is greater. © The two quantities BD Quantity A is greater. D uantity B is greater. © The two quantities are equal. ® The relationship cannot be determined from the ag ral information given, ® The relationship cannot be determined from the x¥0 information given. 4x —5y = 10 14, Quantity A Quantity B 3x + 6y = 22 14t Iyd xts xXx D Quantity A is greater. 16. Quantity A Quantity B ®D Quantity B is greater. 33 xty © The two quantities ®D Quantity A is greater. are equal, Quantity B is greater. ® The relationship cannot ® The two quantities be determined from the are equal. anoeruarienigiven ®D The relationship cannot be determined from the information given. www.grecenternet {emt one 2 oc A(Opp arene ep O11 200 = 01720-55481 gre. Bananas 11nd ck A01677-40978 grecenter@gmail.com Khulna: Rakib Properties, House 11,Sonadanga Main Rd #01940-447162 The diameter of a circle is equal to a diagonal of a square with side 4. Part Three: Quantitative Reasoning. ‘Quantitative Reasoning Practice Set 263 In a three-digit positive integer y, the hundreds digit is three times the units digit. 17, Quantity A Quantity B The circumference 20.2 20. Quantity A Quantity B of the cirde The units digits ofy 4 ® Quantity A is greater. D Quantity A is greater. D Quantity B is greater. ®D Quantity B is greater. ® The two quantities are equal. © The two quantities are equal. ® The relationship cannot D The relationship cannot be determined from the be determined from the information given. information given. ey a 21, What is the average (arithmetic OS mean) of 2x + 3, 5x ~ 4, 6x 18. Quantity A lantity B and 3x - 17 x 0 @ w+4 @D] Quantity A is greater. 3x-2 Quantity B is greater. © wxt2 ® The two quantities are equal. @D x-2 ® The relationship cannot @® 4x42 be determined from the information given. 6(10)" > 60,006 va 19, QuamtityA Quantity B fi ca 1h §: Gg ®D Quantity A is greater. 22. Which of the following statements D Quantity B is greater. D uantity Bis g must be true about the figure © The two quantities are equal. ete ® The relationship cannot be determined from the Seda information ghiGs 6 © a=b @D y=b @® xty=atb ‘www.grecenternet ‘Laima Honve 27, Bleck App Ohanmondl Gov Baye 01931-00040 ‘= 01720-557481 gre ana Hose 12, Road 4 Block A £01677-401978 grecenter@gmail.com Khulna: Rakib Properties, House 11, Sonadanga Maln Rd #01940-447162 Part Three: Quantitative Reasoning 264 Chapter 14 23, if = Zand x + y = 5, which 26. The average (arithmetic mean) es iy ag bowling score of n bowlers is 160. 3 follow ye of the following is greater than y ais wens Ace oes Indicate all possible choices. together with a score of 170 is 161. a What is the number of bowlers, n? v1 dea J bowlers a ws 27. Which of the following are even aw integers? Indicate all possible choices. 24, The product of two integers is 10. The f two odd integers Which of the following could be ) The sum of 7 the average (arithmetic mean) of The product of two odd the two numbers? ee The product of an even Indicate all possible choices. integer and an odd integer (D] The product of two even integers [E] The sum of an even integer and an odd integer 25. Which number is greater than the sum of all the prime factors of 2107 Indicate all possible choices. Wn 28. The circle shown has center T. 5 The measure of angle TVU is os 60°. If the circle has a radius of 3, 19 what is the length of segment RS? OB) a @2 (E] 24 Ss 22 ©3 @33 ® 62 www.grecenter.net e {atmati: Hose 2, Block A(Opp Dhanmondi Gov Boy) #0197 80848 aomossms Ore Banani ou 1, oad 4 ck 0177-4578 Greceneienealicos unc Ral Properties Hose 1, Sonadange Man Re 01340-44262 29. What is the degree measure of angle TRV? @ 4s 60 On ® 105 ® 135 30. There are at least 200 apples in a grocery store, The ratio of the number of oranges to the number 33. of apples is 9 to 10. How many ‘oranges could there be in the store? Indicate all possible numbers. @ om 180 216 (252 @ 3s 31. What is the probability of rolling a 7 with a single roll of two fair dice? 1 Qz 34. 1 QF 2 Qs 1 @; 1 @®, Part Three: Quantitative Reasoning ‘Quantitative Reasoning Practice Set 265 If it takes three days for 10 workers to finish building one house, how many days will it take 15 workers to finish four houses? days In the above square ABCD, the side AB has a length of 4. It is overlaid with a circle, a diameter BC. Which of the following are greater than or equal to the area of the shaded region? Indicate all such amounts. 16 — 16m 16 — 40 16 - 20 O we O wan 1a @ B= AFB and C=C + 3, what is the value of (9) @ 32 pea www.grecenter.net =: 01720-557481 grecenter@gmail.com Glen (S) {Lalmatia: Hosue 21 Block A (Opp Dhanmondl Govt Boys} #01971-080648| ulna: Rak Properties, Houze 11, Sonsdanga Main Rd #01940-442162| ‘Banani: House 121, Road 4 Block A #01677-481878 Part Three: Quantitative Reasoning 266 | chaptersa 35. Rectangle A has a length of 38. Meg is twice as old as Rolf, but 12 inches and a width of 5 inches. three years ago, she was two years Rectangle B has a length of older than Rolf is now. How old is 9 inches and a width of 10 inches. Rolf now? By what number must the area of rectangle A be multiplied in order years old to get the area of rectangle B? 39. The cost, in cents, of manufacturing x crayons is 570 + 0.5x. The crayons sell for 10 cents each, What number of crayons B would need to be sold so that the revenue received is at least equal to the manufacturing cost? Indicate all such numbers. f 50 A c 57 60 36. In right triangle ABC above, side a AB has a length of 5, while side BC has a length of 13. What is the Hl 95 ‘number of square units for the area of ABC? ararennit @ 21 37, If the average test score of four pa students is 85, which of the L following could the fifth student © wt score in order to have the new Diss average be greater than 84 and yy less than 86? a @®i-1 Indicate all such scores. Al 8s 86 85 OB f 80 ‘www.grecenter.net {aimatia Hose 2, Block (Opp Dhanmand Govt Boy 01921-080648| rinessne GLE eee era (grecanfare niall ulna Rat Proper House 1, Sonadangs Man a 01940-487162 Questions 41-45 are based on the following graphs. Total # Table Sows Sold finer] Tota Profit fom Table Saw Sales (in dlls) ~AALEL Breakdown of Profits in 1998 Lathes 58 oi ape reson tre i 41, In 1998, what were the total profits from sales of all the hardware tools? Part Three: Quantitative Reasoning Quantitative Reasoning Practice Set | 267 42. Which year had the greatest percentage increase in number of table saws sold from the previous year? ® 1993 1995 © 1997 @D 2000 ®D 201 43. Of the following, what is the closest to the percentage change in profits from table saws between 1998 and 19992 ®D A 50% increase A 33% increase D A 17% decrease D A 33% decrease D A50% decrease 4. If the fixed cost of manufacturing table saws in 1993 was $22,000, how much did each table saw sell for? $s 45. In 1998, what were the approximate profits from the sales of cordless drills? ® $50,000 ® $70,000 ©® $80,000 ®D $90,000 ® $100,000 wmwgrecenternet assis ea fpe Shien oa oven ‘= 01720-557481 0-5; OLE. (#) iautiatnuantin finseeannnascial grecenter@gmail.com Banani: House 121, Road 4 Block A.401677-481878 268 Part Three: Quantitative Reasoning ‘Chapter 14 Questions 46-49 are based on the following graph and table. ‘Water age by Yet Twn W along (Hsceat enor] Daily Water Usage Statistics (with Efficient Appliances and Good Maintenance) Use Gallons per capita Showers 9 Clothes Washers 10 Toilets 8 | Leaks 4 Faucets im Other 4 46. Which best describes the range (in billions of gallons) for residential water consumption for the time period indicated? @® wo 20 © 30 @®D 40 @® 50 47. 48. For the year in which total consumption exceeded residential ‘consumption by the least number of gallons, approximately what percent did the residential usage comprise of the total usage? 68% 75% 88% 95% 98% 608008 In 2004, only 10,000 residents lived in homes with efficient appliances and good maintenance of the plumbing. How many gallons per day were used by these residents for the three purposes requiring the most water? 110,000 160,000 270,000 300,000 460,000 08608 grecenter@gmail.com Banani: House 121, Road 4 Block A.#01677-481878 waregrecenternet {aati Sle pp Dhaene Boye 0102 = omosiue (GLE, [S) 0-5 Khulna Rak Properties House 11,Sonedanga Main Rd #01940-442162_ 49. Households with efficient appliances and good maintenance of plumbing fixtures can reduce water consumption by about 35%. If approximately half of the residential consumption in 2010 ‘was by households with these characteristics, how many billions of gallons of water were saved that year? @s @Du @ 40 @D 2 Ds 51 52. Questions 50-56 are based on the following graph. Enrollment at Three Community Colleges, 1990-2005 (in hundreds) 7 50 40 30 20 10. 33. MT] ° 1990 1995.” 2000 2005 Vall Part Three: Quantitative Reasoning (Quantitative Reasoning Practice Set 269 What was the total enrollment (in hundreds) for the three colleges in 2000? 10 125 135 140 150 08008 Which enrollment showed the greatest increase over the years shown? ® Valley between 2000 and 2005 ® Greendale between 1995 and 2000 ]® Greendale between 2000 and 2005 @® Eastern between 1995 and 2000 ®@ Eastern between 2000 and 2005 Expenses at the three colleges have shown the same percent increase between 2000 and 2005 as the percent increase in 50. For the year in which the ratio was enrollment, Approximately what the highest, what was the ratio of is that percent? the alae Greendale to the @® 10% Valley? enrollment at Valley aa @® ei © 27% ae D 36% ou ® 40% @ 3 @ a ‘www.grecenter.net Lama: Hose 2, Block A(Opp Dhanmend Gov Boy) #0197 one =omossmn Gre. aan we 1, ond ck s01677 49378 grecenter@gmail.com ‘ulna Rai Proprio House 1, Sonadangs Man Re 01940447162 Part Three: Quantitative Reasoning 270 | chapter14 54, To the nearest hundred, what was Question 57-60 refer to the following the average (arithmetic mean) graphs: enrollment in 1995 at the three colleges? “Team Revenues for 1997 @® 1,000 190 By Ober Reenve 1,500 01 GD Media Revenue oe sa 1 Venue Revenue @D 80,000 tone et PS) ® 85,000 Sistas) « el (ase] 55. Valley and Greendale have plans « eS eK. to combine into one college in the » = & future to make better use of their oe resources. If the 2005 combined student to faculty ratio of remains the same, how many faculty members will be needed for the new college, based on current enrollment? faculty 56. The projected enrollment at Eastern in the next academic year is 6,200 students. What percent increase will that be over the enrollment in 2005? Percentages of Venue Revenues for Team X, 1997 D Otter 1 Merchandise DD Ticket sles Bi Food sates Ty Beer sales 57. For the team with the median D®D 12% amount of venue revenue for ® 15% 1997, media revenue represented D 1% approximately what percent of that D 20% team's total revenue for that year? BD u% @® 25% D® 30% @ 40% D 55% ® 60% wwwgrecenternet ‘mata: Honus 27, Block A(Opp Dhanmondi Govt oy 0192-08004 = 01720-557981 Oren Banani: House 121, Rand Block A 00167740197 grecenter@gmail.com Khulna: Rab Properties Mouse 11,Sonadanga Maln Rd #01940-447162 Part Three: Quantitative Reasoning, ‘Quantitative Reasoning Practice Set 21 58. Of the following, which is greater 60. If Team Y earned a total revenue than the amount of revenue, in of $150 million or greater in 1998, millions of dollars, earned by ‘Team Y's total revenue increased ‘Team X through food sales in by approximately what percent 1997? from 1997 to 1998? Indicate al] such amounts. Indicate all such percents. @7 [Al 20% B) 25% 30% DB is DB] 35% Hl 2 ] 40% 59. In 1997, which teams had media revenues less than $25 million? Indicate al] such teams. ) Team X Team Y Team Z (8) TeamQ ©) TeamR worw.grecenter.net gy {ata Hosue 2, Block A(Opp Dhanmeond Govt Boy) #0197 o80848 57481 ‘Banani: House 12, Road Block A#01677-481978 = 017205; Geen pater etait aeamneleneceal grecenter@gmail.com Part Three: Quantitative Reasoning aa | Gamera ANSWERS AND EXPLANATIONS 1G 16. A 3. B 46. C 2¢ 17. B 32. 8 47. 3D 18. D 33. CDE 48. D 4a 19. D 34.5 49.:B 5. B 20. B 35. 15 50. B 6A 21. D 36. 30 SLA ZG 2 E 37. ABCD 52. D BA 23. B, 38. 5. 53. C 9A 24. ABE 39. CDE 54. C 10. A 25. CDE 40. A 55. 1000 nc 26. 9 41. 200,000 56. E 12. ¢ 27. A.C,D aA 57. € 13. D 28. C 43. D 58. DE 4, D 29. D 44. 700 59 E 15. B 30. BC,D,E 45. B 60. BLC,D.E Diagnostic Tool Tally up your score and write your results below. Total Total Correct: __out of 60 correct By Question Type Quantitative Comparisons (questions 1-20)____ out of 20 correct Problem Solving (questions 21-40) ____out of 20 correct Data Interpretations (questions 41-60)_____ out of 20 correct DIAGNOSE Your RESULTS Look back at the questions you got wrong and think about your experience answering them. © STEP1 Find the Road Blocks Ifyou struggled to answer some questions, to improve your score you need to pinpoint ‘exactly what “road blocks" tripped you up. To do that, ask yourself the following two questions. ‘www.grecenter.net {matin Honus 2, Block (Opp Dhanmondi ov Boyt 40192-08048 ‘=: 01720-557481 ore. SananiHowe 12, Row lock A#01677481573 precantar@ pe alten ulna Ral Properties, House 11,Sonadanga Main Rd #01940-442162| Part Three: Quantitative Reasoning ‘Quantitative Reasoning Practice Set ‘Am | weak in the skills being tested? This will be very easy for you to judge. Maybe you've forgotten how to figure out the atea of a triangle or what PEMDAS stands for. If you know you need to brush up on. your math skills, try the Kaplan GRE Exam Math Workbook, which contains a focused review of all the fundamental math concepts tested on the GRE, as well as practice exercises to build speed and accuracy. Was it the question types that threw me off? Then you need to become more comfortable with them! Quantitative Comparisons have a unique format, and Data Interpretation questions can be daunting with their charts, graphs, and tables. If this was a problem for you, go back to the beginning of this chapter and review the Kaplan principles and methods for the question types you struggled with. Make sure you understand the principles and how to apply the methods. These strategies help to improve your speed and efficiency on test day. Remember, it's not a math test; it's a critical-reasoning test. Also, get as much practice as you can so that you grow more at ease with the ques- tion formats. For even more practice, try the Kaplan GRE Exam Math Workbook, which includes practice sets for each question type. © STEP2 Find the Blind Spots Did you answer some questions quickly and confidently but get them wrong anyway? When you come across wrong answers like these, you need to figure out what you thought you were doing right, what it turns out you were doing wrong, and why that happened. The best way to do that is to read the answer explanations! The explanations give you a detailed breakdown of why the correct answer is correct and why all the other answers choices are wrong. This helps to reinforce the Kaplan. principles and methods for each question type and helps you figure out what blind- sided you so it doesn’t happen again. Also, just as with your “roadblocks,” try to get in as much practice as you can. © STEP 3 Reinforce Your Strengths Now read through all the answer explanations for the ones you got right. Again, this helps to reinforce the Kaplan principles and methods for each question type, which in tur helps you work more efficiently so you can get the score you want. Keep your skills sharp with more practice. www.grecenter.net =: 01720-557481 grecenter@gmail.com Geen Lalmati: Hosue 211 Block A(Opp Dhanmondi Govt Boye! 401921000848 | ‘Banani: House 121,Road 4, Block A 01677-48197) ‘ulna: Rakib Properties, House 11,Senadanga Main Rd #01940-447163| 273 274 Part Three: Quantitative Reasoning Chapter 14 {As soon as you are comfortable with all the GRE question types and Kaplan methods, complete a full-length practice test under timed conditions. Practice tests serve as milestones; they help you to chart your progress! So don't save them all for the final weeks before your test day. For even more practice, you can also try the Kaplan GRE Quiz Bank. You get more than 1,000 questions that you can access 24/7 from any Internet browser, each with comprehensive explanations. You can even customize your quizzes based on question type, content, and difficulty level. Take quizzes in Timed Mode to test your stamina or in Tutor Mode to see explanations as you work. Best of all, you also get detailed reports to track your progress. Visit kaplanGRE.com for more details on our Quiz Bank and for more information on our other online and classroom-based options. If the Quantitative Reasoning section is where you need the most help, you should check out the new Kaplan course, GRE Math Advantage—Anywhere. It’s available online anywhere you are, and you can get live instruction from an expert Kaplan GRE teacher. 4, 6 Quantity A is a permutation because order matters. The number of ways 6 people can be arranged in a line of 3 people where order matters is 6 x 5 x 4 = 120. Quantity 8 is a combination because order does not matter. The number of ways 10 people can be 1ol___ 10x9x8 arranged ina line of 3 people where order does not matter is .C, = spqjogy = WPS 720 — 120, 1he two quantities are equal all There are three sets of vertical angles in this diagram: (ad), (b,e), and (¢ f).In Quantity A, you can substitute b for e because they are vertical angles and therefore equa; this leaves, the sum a + 6 + cin Quantity A. Because these are the three angles on one side of a straight line, they sum to 180. Similarly, after substituting e for b in Quantity 8, b+ d+ f is the same thing as d + e + f,or also 180. The two quantities are equal. 3D Pick a value for p and see what effect it has on rand s. fp = 1, = (7 X 1) +3 = 10, and 5 = (3X 1) + 7 = 10, and the two columns are equal. But if p = 0, r= (7 X 0) +3 = 3, and s = (3 0) +7 = 7, and Column A is less than Column B, Because there are at least ‘two different possible relationships, the answer is (D). 4k Use the Picking Numbers strategy to answer this question. Suppose the original selling price of the shirt, x, is $100. After a 10% increase in price, the shirt would sell for 110% of $100, which is $110. If there is a 10% decrease next, the shirt would sell for 90% of the cur- rent price. That would be 90% of $110: 0.9 x $110 = $99. This price is less than the original amount, x, so Quantity A is greater. www.grecenter.net 2 01720-557481 grecenter@gmail.com ‘OfScn LalmatiscHosue 2/1, lock (Opp Dhanmondi Govt Boy 401921-080848 ‘Banani: House 121, Road 4, Block A.#01677-481978| Khulna: Rakib Properties Mouse 11,Sonadanga Main Rd #01940-442162 Part Three: Quantitative Reasoning (Quantitative Reasoning Practice Set 5. B Try to set the quantities equal. Ifxis 50, then the bookstore started out with $0 dictionaries. Ten J of them were purchased. You can see already that the quantities cat be equa, 9f 50 won't yield an integer, But go ahead and see whether the answer is (A) or (B). Because of 50s close to 6, after these dictionaries were purchased, the store would have been left with about 50 — 6 or 44 dictionaries. Then it received 10 more, giving a total of about 54 dictionaries. But this is more than the store actually ended up with; it only had 52. Therefore, it must have started with fewer than 50 dictionaries, and Quantity B is 32, Quantity A is larger. 7. B The diagonal of a square of side 4 is 4,2. The circumference ofa circle ism times the diam- eter. So, the circumference of this cicle is 4:2. Now write Quantity B, 20/2, as 4(5)\2 and you can compare the quantities piece by piece. The factors of 4 and 12 are the same in both quantities, but = is less than 5. So, Quantity B is larger. 18. D You could Pick Numbers here or else just use logic. You know that z is positive and that xand y are less than z. But does that mean that x or y must be negative? Not at all—they worw.grecentennet {atmata: Honus 2, Bloc (Opp Dhanmondi Gov Boy #0192 00048 = 01720-551481 gre... Banani:Howe 12, Road lock £01677 481573 grecenter@gmail.com nna taki roperin House 1 Sonedanga Man Re ot 9a0-471€2 Part Three: Quantitative Reasoning ‘Quantitative Reasoning Practice Set could be, but they could also be positive. For instance, suppose x = 1, y = 2, and2= 3. Then Quantity A would be larger. However, ifx = —1, y= 0, and 2 = 1, then Quantity B would be larger. You need more information to determine the relationship between the quantities. The answer is (D). 19. D Divide both sides of the inequality by 6. You're left with (10)* > 10,001. 10,001 can also be written as 10 + 1, so you know that (10)" > 10 + 1. Therefore, Quantity A, n, must be 5 or greater. Quantity B is 6. Because n could be less than, equal to, or greater than 6, you need more information. 20. B Try to set the quantities equal. Could the units digit of y be 4? If itis, and the hundreds digit is three times the units digit, then the hundreds’ digit must be . .. 12? That can't be right. A digit must be one of the integers 0 through 9; 12 isn't a digit. Therefore, 4 is too big to be the units digit of y. You don't know what the units digit of y is, but you know that it must be less than 4. Quantity 8 is greater than Quantity A. 21. D To find the average, add the quantities together and divide by 4, (2x + 3) + (5x — 4) + (6x — 6) + Bx ~ 1) = 16x — 8 and 1X8 GB = 4 — 2. The correct choice is (D). 22.E GIe When a transversal cuts a pair of parallel lines, in this case ¢, and ¢, the angles are always supplementary and their sum is 180. So, the sum (x + y) is equal to the sum (a + 6). The exact values of the individual angle measures cannot be determined from the figure. The answer is (E). 23. B,C,D,E UF = 5, then 3x = 2y and y = 3. Substitute y = 3 into the equation x + y = 5.x + 3 = 5,2x + 3x = 10,5x = 10,x = 2. Then, y = 3% = 32) ~ 3 and y? = 9. 50 any number greater than 9 under the radical will be greater than y. Therefore, the correct choices are (8), (C), (D), and (€) ‘www.grecenter.net = 01720-557481 grecenter@gmail.com {mati Hosue 21, Block A (Opp Ohanmond Govt Boys) 401971-000848) Banani: House 121, oad 4, Block A.£01677-481978| Khulna: Rakib Properties, ouse 11, Sonadanga Main Rd #01940-447162 279 Part Three: Quantitative Reasoning 280 | chapters4 24. A,B, E The best place to start here is with pairs of positive integers that have a product of 10. The umbers 5 and 2 have a product of 10, as do 10 and 1. But remember that integers may benega- tive, so —1 and —10 are possible, as well as ~2 and —5. The mean of —1 and —10 is 5.5; the mean of ~2 and ~5 is ~3.5. The mean of 2and 5 is 35. The correct answers are (A), (B) and (E). 25. C,D,E The prime factorization of 210 is 2 x 3 x 5 X 7, The sum of the prime factors is 2+ 3 + 5 +7 = 17,So, the correct choices are (C), (D), and (E). 26. 9 Use the definition of average to write the sum of the first n bowlers’ scores: um of scotes — average and therefore, n X average = sum of scores. Substitute the values given in the question and you have 160n = sum of scores for the initial set of bowlers. Now write the formula for the average again, using the additional score of 170. Now there aren + 1 bowlers sum of scores 160n + 170 n+ average 161 Cross multiply and use algebra to solve for n. 160n + 170 = 161(n + 1) 160n + 170 = 161n + 161 170 — 161 = 161n ~ 160n a There were 9 bowlers in the original group. 27. ACD Test each answer choice by Picking Numbers; use 2 for the even number and 3 for the odd number. The results you get using these numbers will be representative of all integers. Sum of two odd integers:3 + 3 = 6 (even) Product of two odd integers: 3 x 3 = 9 (odd) Product of an even integer and an odd integer: 2 x 3 = 6 (even) Product of two even integers: 2 x 2 = 4 (even) ‘Sum of an even integer and an odd integer: 2 + 3 = 5 (odd) Choices (A), (C), and (D) are correct. www.grecenternet {aimatn Honus 2B (Opp Dhanmondi Gov Boyt #01921 0004 = 01720-557481 ore. Sanani:Howe 12%, Rou lock A #01677-481573 (qrecentar@ oa Talizoen Khulna Rak Properties, House 11,Sonsdangs Main Ré#01940-442162| Part Three: Quantitative Reasoning ‘Quantitative Reasoning Practice Set 28. ¢ ‘There are several steps involved with this problem, but none is too complicated. The circle has its center at point T. Start with the triangle on the right whose vertices are at T and ‘two points on the circumference of the circle. This makes two of its sides radii of the circle Because all radii must have equal length, this makes the triangle an isosceles triangle. In addition, you're told one of the base angles of this triangle has measure 60°. Thus, the other base angle must also have measure 60° (since the base angles in an isosceles triangle have equal measure), The sum of the two base angles is 120°, leaving 180 — 120 or 60° for the other angle, the one at point T making ATUV an equilateral triangle). Now, angle A7S is opposite this 60° angle; so, its measure must also be 60°. Therefore, ARST is another equilateral triangle, and its sides are 3. Therefore, the length af RSis 3, choice (C). 29. D First find the value of x, using the fact that there are 180" in a straight line. Set the sum of, the angle measures equal to 180: 5x + 4x + 3x = 180, 12x = 180, and x = 15. Angle TRV equals 4x + 3x = 7x, which is 105°. Choice (0D) is correct. 20. 8.6.0.8 You know that the ratio of oranges to apples is 9 to 10 and that there are at least 200 apples. The ratio tells you that there are more apples than oranges. At the minimum, there must be 10 oranges o sai he proportion 2 = 180 There could be more than 20 apple, ‘so any number of oranges greater than 180 for which the ratio 9:10 applies is correct. All of the choices are multiples of 9, so the correct choices are (B), (C),(D), and (E). 31. B ‘The probability formula is, Number of desired outcomes Probability = Number of possible outcomes When one die is rolled, there are six possible outcomes. When two dice are rolled, the number of possible outcomes is 6 x 6, of 36. Getting a total value of 7 can be achieved in the following ways: (1, 6), (2, 5, (, 4) (4, 3), (5, 2), and (6, 1), There are six possible ways. So the probity of ling a toa of 7 i$; which canbe reduced to choice (8 na. 8 In the first scenario, each day, Bouse 3 days Fof the house will be built. Because there are www.grecenter.net re 1 ‘are moan Olen Lalmti: Hose 211, Block A (Opp Dhanmondi Govt Boys! 401921-000848| ‘Banani: House 121, Road 4, Block A.#01677-481978| Khulna: Rakib Properties Howse 11, Sonadanga Main Rd #01940-442162 281 Part Three: Quantitative Reasoning 282 | chaptersa 10 workers, each person can build > of a house each day. In the second scenario, there are 15 workers, so that means 15 x a = z a house can be built each day, Four houses could, therefore, be builtin 8 days: 4 POUseS_ = 4 x 2 = 8 days. Yhouse/day 33. CDE ‘The area of the shaded region is the area of the square minus the area of the portion of the circle that is inside the square. The area of a square is its side squared. The area of square ABCD is 4* = 4 X 4, which is 16. Now find the area of the portion of the circle that is inside the square. Because the diameter of the circle is a side of the square, you know that exactly one-half of the citcle's area is inside the square. Because the diameter of the Circle is twice the radius, the radius of the circle is 4, or 2, The area of a circle with a radius ‘ris nr®, The area of the complete circle in this question is 72°), which is 4. So half the area of this circle is 27. Thus, the area of the shaded region is 16 — 2x. That means that 16 — 4 and 16 — 16x are less than 16 ~ 2x, so they cannot be correct choices. However, the sum of 16 and any positive number is greater than 16 and also greater than 16 — 2r. So, the correct choices are (C), (D), and (E). 34. 5 Let's first find the value of 94 Then we'll find the value of (94) @ 3. ince C# = C+ 3,9H=943= 12, Then (94) #3 = 1203. ince are =Bf3-5 Since A@ 8 = 448, 123 = 243-15 Thus, (94) @3=5. 35. 15 The area of a rectangle i its length times its width. The area of rectangle A is 12 x 5 = 60 The area of rectangle B is 9 x 10 = 90. So the area 60 of rectangle A must be multiplied by a number, which you can call x, to obtain the area 90 of rectangle 8. 2 cores Then 60x = 90, So x= 20 = 3 = 1. 36. 30 Here's a problem where it really pays to learn the special right triangles. Because one leg of the right triangle is § and the hypotenuse is 13, you have a special right triangle, the 5:12:13 right triangle, So the length of ACis 12. The area of a triangle is + of the base times the height. The area of a right triangle is 3.x (lea), X (leg),, because one leg can be considered to be the base and the other leg can be considered to be the height. So the area of triangle ABC st X (AQ) x (AB) = 3x 12 X 5 = 6 X 5 = 30, The answer is 30, ‘www.grecenter.net lock A (Opp Dhanmondi Govt Boy! 40192-80648 i rare (#) : ran House 121,Road 4 Block A 401677481878 eae OSes (BE) a taarennr attndme wir Part Three: Quantitative Reasoning ‘Quantitative Reasoning Practice Set 37. A,B,C.D ‘The average formula is as follows: ‘Sum of the terms ‘Number of terms average Therefore ‘Sum of the terms = Average X Number of terms The sum of the scores of the four students whose average was 85 is 85(4) = 340. Let's call the fifth students score x the new average ito be greater than 84 and les than 86 and the sum of the sores ofa fv students is 240-+ x then 84 < 392+ < 96. you multiply all parts of the inequality by 5, you get 420 < 340 + x < 430. Subtracting 340 from all parts of the inequality, you get 80 < x < 90, making (A), (B), (C), and (D) the correct choices. 38. 5 This question can be broken into two equations with two unknowns, Meg's age now (M) and Rolf ’s age now (A). Equation i shows the relationship now; equation ii shows the relationship 3 years ago. iM=2xR WLM-3=R42 Substitute 28 for M in equation ii and solve for R: M-3=R+2 2R-3=R+2 2R-R=243 R=5 Rolf is 5 years old now. 39. CDE The cost of manufacturing x crayons is (570 + 0.5x) cents. Because each crayon sells for 10 cents, x crayons will sell for 10x cents. You want the smallest value of x such that 10x ‘cents is at least 570 + 0.5x cents. So you must solve the inequality 10x = 570 + 0.5x for the range of values that x can have. 1x = 570 + 0.5x 95x = $70 95x = 570 x= 60 ‘The minimum number of crayons is 60. So, selling 60, 61, or 95 crayons would also make the revenue received at least equal to the manufacturing cost. A great strategy to use here would be Backsolving, sta 19 with either (B) or (D). worm grecenternet cents ena ha eps apanlgt ommaal somos OLE Sanotee stadt grea grecenter@gmail.com ulna: Ral Properties, House 11,Senadanga Man Rd #01940-442162| 283 Part Three: Quantitative Reasoning 284 | chaptere 40. A You can write that 15% = 31 — 2. Canceling a factor of x from the numerator and denominator of , you have % = 1 ™ wy Nyt ayy ay Ty The answer is (A. 41. 200,000 From the second bar graph, the profits from table saws in 1998 were $60,000. From the pie chart, table saws were 30% of the total profits. Let’ call the total profits T dollars. 30% of T dollars is $60,000. So 0.37 = 60,000, and 60,000 _ 60,000 _ 10 x 60,000 _ 600,000 03 em 3 3 10 200,000. 42k The year with the biggest percent increase over the previous year will be the year in which the increase Is the biggest fraction of the amount from the previous year. Notice that in 1993, the increase from 1992 was approximately 60 — 30, or30. This is approximately a 100 percent increase, and it is the greatest percent increase over the previous year among all the years from 1993 through 2001. There was a greater increase in number of table saws from 1996 to 1997 than from 1992 to 1993, about 110 — 70 = 40. However, the percent incease from 199610 1957s approximately £2 100% which sles than 100%, so choke (A) is correct. 43. 0 In 1998, the profits from table saws were approximately $60,000. In 1999, the profits from table saws were approximately $40,000. From 1998 to 1999, there was a decrease in the profits from table saws. In general, Original value — New value p he ee Original value 100% Here, the percent decrease is approximately $60,000 ~ $40,000 $20,000 360000 * 1% = “69000 A percent decrease of 33% is closest to (D). X 100% = 4 100% = 2336 44, 700 In 1993, the profits were $20,000. Using the formula Profit = Revenue — Cost, you can write Revenue = Cost + Profit. The cost was $22,000, So the revenue was $22,000 + $20,000 = $42,000. Because in 1993, 60 table saws were sold, each table saw was sold for $42,000 60 » which is $700. wrwgrecenternet {atte en 27 SecA Dhnancedl Govt Boy) OI cou =omosoms OTe, anion 1, fond 4 lek A467 49978 grecenter@gmail.com Khulna: Rak Properties, Houce 11, Sonsdanga Main Rd #01940-442162| Part Three: Quantitative Reasoning ‘Quantitative Reasoning Practice Set 45, B In 1998, the profits from table saws were about $60,000, and this profit was 30% of the total profits. Let's call the total profits T dollars. Then 30% of T dollars is $60,000. So 60,000 10 x 60,000 _ 600,000 60,000 _ Jo 0.37 = 60,000, and 7 = $08°° — 60,000 x19 $ profits in 1998 were approximately $200,000. The profits from cordless drills were 35% of the total. So the profits from cordless drils were approximately 0.35($200,000), which is $70,000 or answer (B). 100,000. The total 46. ¢ The residential usage (in billions) in 2000 was about 22; the usage was about 52 in 2010. The range is the difference because the residential usage increased over the time period. 52 ~ 22 = 30, so the range is about 30 billion gallons. The correct answer is (C). a7. The two amounts were closest to each other in 2002. The residential amount appears to bbe about 28; the total appears to be about 32: 28 + 32 = 0.875. Choice (C) is the closest. 48. D ‘The three usages with the greatest amounts per person are faucets, washers, and showers totaling 30 gallons per day. Multiply by 10,000 to get 300,000, choice (D). 49. B ‘The residential consumption (in billions) in 2010 was approximately 52. Take half of that amount, 26, to represent the amount of water used by households with efficient appli- ances and plumbing. Let W represent the amount of water these households would have used otherwise. Set up a percent equation to solve for W., Remember, the savings were 35%, so subtract 35 from 100 to find the amount that would have been used. 26 = (100% — 353%) x W 26 = 65% x W 26 = 065 x W 26 i wn 40-W The savings in billions of gallons were 40 ~ 26 = 14. The correct answer is (B) 50. B Compare the bar for Greendale to the bar for Valley for each year shown. The greatest dif- ference occurred in 1990, Write the ratio 50:10 and reduce to 5:1. The correct answer is (B) 51. Combine the three values shown for the year 2000: 15 + 55 + 40 = 110. Choice (A) is correct. A ‘www.grecenter.net = 01120-557481 grecenter@gmail.com Ore... (#) Lalmatia: Horwe 2/1, lock (Opp Dhanmondi Govt Boy! #01921-080648 ‘anan:Houze 121, Road 4, Block A.#01677-481978| Kul: Reb Properties, House 11,Senadanga Main Rd #01940-442162 285 286 Part Three: Quantitative Reasoning Chapter 14 52. D Check each option carefully, being sure to look at the correct bar and the correct time frame. This is the most efficient way to answer the question. Except for choice (D), each increase was 1,000 students. The correct answer is (D) because there was an increase of 2,000 students in that time frame. 53. C To find the percent increase, first find the total enrollment in 2000 and the total enroliment in 2005. Don't worry about the hundreds; just use the numbers from the graph. For 2000, this number is 110. For 2005, this number is 140. To find the percent increase, compare the amount of increase, 30, to the original amount, 110, The closest answer is (C). 54, C The three enrollment figures for 1995 are 1,500 + 4,500 + 2,000 = 8,000. Divide by 3 to find the average: 8,000 ~ 3 ~ 2,667. The clasest answer choice is (C), 2,700. 55. 1000 There were 2,500 + 6,500 = 9,000 students enrolled in 2005. To maintain the 9:1 ratio, 1,000 faculty will be needed. 56. E The enrollment at Eastern in 2005 was 5,000, so an increase of 1,200 students is expected, Compare 1,200 to 5,000 or simply compare 12 to 50. 12 is to 50 as 24 is to 100, so the percent increase is 24%. The correct answer is (E). or Before you answer any graph question, begin by examining the graphs. Here you have two graphs, a segmented bar graph representing team revenue breakdowns for five teams and a pie chart showing the distribution of venue revenues for Team X. You're now ready to attack the question, which asks you to find the team with the median venue revenue for 1997 and to determine what percent of that team’s total revenue is media revenue. This question must refer to the first graph, and the first part of the question—finding the team with the median venue revenue—is a simple matter. Median refers to the number in the middle. Looking at the white portions of the bars in the top graph, you see that Team Z has the median venue revenue, The fastest approach to the answer here (and throughout graph questions generally) is to approximate. The downside to bar graphs is that it’s often very hard to get a read on the values. The upside is that if you approximate, often you don’t have to read the values. Here you need to determine what percent of Team 2's bar is represented by media revenue (the segment in the middle—always be especially care~ ful to isolate the correct piece of data). By approximating, you should be able to see that the middle segment is more than a third and less than a half of the entire bar. Thus the correct answer has to be between 33% and 50%. The only answer that works is (C), 40%, www.grecenternet 01720-957481 grecenter@gmail.com 7 e Latmatl: Hose 21, Block A (Opp Dhanmondi Govt Boys) #01921-080848| ‘Banani House 121,Road 4 lock A 01677-481973) Khulna: Rak Properties, House 11,Sonsdanga Main Rd #01940-442162| Part Three: Quantitative Reasoning Quantitative Reasoning Practice Set | 287 58. DE The key to this question is that it involves both graphs. The question asks for the amount Team X earned through food sales, which takes you first to the pie chart, where you see that food sales accounted for 35% of the venue revenues for Team X. But to convert that to a dol- lar amount, you need a figure for the amount earned in venue revenues by Team X in 1997. According to the bar graph, this is somewhere around $40 million. Now, take 35% of $40 mil- 0.35 X 40 = 14, so the answer is any amount greater than 14. The answers are (D) and (E). 59. E Look at the graph; team Q had media revenue of 35 — 5 = 30 million, and teams X, Y, and Z had media revenues greater than team Q. Team R had media revenue of 30 ~ 10 20 million. The only correct choice is (E). 60. BC,DE Percent change problems are extremely popular graph questions, and as long as you set them up correctly, they're generally no problem. This question asks for the approximate percent increase in Team Y's total revenue from 1997 to 1998, so you need to figure out (roughly) the amount of increase, place that over the original (or smaller) amount, then convert the fraction into a percent. You are given the total revenue for 1998 as $150 million oF greater, so you need to locate the total revenue for 1997 from the bar graph. It looks to be approximately $120 million, so the amount of increase is $30 million, and the original (or smaller) amount is $120 million. Now let’s apply the formula: $30 million 3120 millon 1 Percent Increase 100% X 10096 a = 25% So, any percent greater than or equal to 25% Is the answer. The answers are (B), (C), (D), and (E), QUANTITATIVE REASONING PRACTICE SET 2 NumBers All numbers are real numbers. FiGURES The position of points, lines, angles, and so on, may be assumed to be in the order shown; all lengths and angle measures may be assumed to be positive. Lines shown as straight may be assumed to be straight. wonw.grecenternet ae aaa ee eae eee ie omossnet OLE, Seccmun calaasrkicter aan grecenter@gmail.com Kul: Reb Properties House 11,Sonadanga Main Rd #01940-442162 Part Three: Quantitative Reasoning 288 | chapters Figures lie in the plane of the paper unless otherwise stated. Figures that accompany questions are intended to provide useful information, How- ever, unless a note states that a figure has been drawn to scale, you should solve the problems by using your knowledge of mathematics, not by estimation or measurement. DIRECTIONS Each of the following questions, 1-20, consists of two quantities, Quantity A and ‘Quantity B. You are to compare the two quantities and choose @® if Quantity A is greater; if Quantity B is greater; © if the two quantities are equal; or D if the relationship cannot be determined from the information given. COMMON INFORMATION In a question, information concerning one or both of the quantities to be compared is centered above the two quantities. A symbol that appears in both quantities represents the same thing in Quantity A as it does in Quantity 8. 1. Quantity A ‘Quantity B x>0 ‘The number of |The number of Positive integer positive integer 2. Quantity A Quantity B factors of 96 factors of 72 x+1 7 x e+ @ Quantity A is greater. uantity r Quanity B is greater, eee juantity B is greater. © The two quantities are equal ioe x ae , ® The relationship cannot binant be determined from the @D The reatiomship Gatos information given. be determined from the information given. ‘www.grecenter.net 080848 7-401978| ee ee ore. eer ae eee = 01720-537481 pan 3. Quantity A Quantity B ? 4 D Quantity A is greater. D Quantity B is greater. © The two quantities are equal. @D The relationship cannot be determined from the information given. 4. QuantityA Quantity B The number The number of seconds in of hours in 7 hours 52 weeks ®D Quantity A is greater. Quantity B is greater. © The two quantities are equal. ® The relationship cannot be determined from the information given 5. QuantityA Quantity B D Quantity A is greater. Quantity B is greater. © The two quantities are equal. ® The relationship cannot be determined from the information given. Part Three: Quantitative Reasoning Quantitative Reasoning Practice set | 289 x>2 6. Quantity A Quantity B “ 4x ® Quantity A is greater. Quantity B is greater. © The two quantities are equal. ® The relationship cannot be determined from the information given. 7. QuantityA Quantity B (@+2@-2 @+Na-H D Quantity A is greater. D Quantity B is greater. ]& The two quantities are equal. ®D The relationship cannot be determined from the information given. 8. Quantity A Quantity B The area ofa Twice the area square with of a square with perimeter 16 perimeter 8 ® Quantity A is greater. ®D Quantity B is greater. © The two quantities are equal. ® The relationship cannot be determined from the information given. {Lalmatia: Hosue 21 Block A (Opp Dhanmondl Govt Boys} #01971-080648| www.grecenter.net Se ie 2) grecenter@gmail.com Part Three: Quantitative Reasoning 290 | chapters ‘The area of one face of a cube is 35. 8 9. Quantity A Quantity B The total The total surface area _volume of the + 7 of the cube. cube. pate D Quantity A is greater. Quantity B is greater. 11. Quantity A Quantity B © The two quantities are equal The area of The area of @® The relationship cannot ABAD ABCD be determined from the informs gfyen, @ Quantity A is greater. ®D Quantity B is greater. ® The two quantities are equal. ®D The relationship cannot be determined from the x x information given. tity A is greater. = a) " 2 12. QuantityA Quantity B D Quantity B is greater. preven irene © The two quantities are equal D Quantity A is greater. ® The relationship cannot Quantity B is greater. be determined from the © The two quantities information given. sre gull ®D The relationship cannot be determined from the information given. www.grecenter.net Laman Hosue 2, Block (Opp Dhanmondi Govt Boy 01921-08088 01720-557481 ore (| BanancHouse 121, Rous lock A #01677481878 = 01120-5% Khulna: Rakib Properties, House 11, Sonadanga Main Rd #01940-442162| grecenter@gmail.com A circle with radius r is inscribed in a square, ris greater than or equal to 1 13, Quantity A Quantity B The area of r the square Rot in the circle D Quantity A is greater. Quantity B is greater. ®D The two quantities are equal ® The relationship cannot be determined from the information given. Eight points lie on a circle. M4. Quantity A Quantity B The number The number of triangles _of pentagons you can you can make make using _using 5 points. 3 points as as vertices vertices ®D Quantity A is greater. Quantity B is greater. ® The two quantities are equal. ® The relationship cannot be determined from the information given. 15. 16, Part Three: Quantitative Reasoning ‘Quantitative Reasoning Practice Set 291 ‘Two die are rolled. Quantity A Quantity B The number The number of of different different ways ways to geta to get a sum sum of 5 of 9 D Quantity A is greater. @D Quantity B is greater. ® The two quantities are equal. ®D The relationship cannot be determined from the information given. QuantityA Quantity B ‘The number The number of of different _ different ways ways to to arrange the arrange the letters in the letters in the word circle word square ®D Quantity A is greater. Quantity B is greater. © The two quantities are equal ® The relationship cannot be determined from the information given. www.grecenternet {ante on 27sec App haanondl Gort Boy) O10 = 01720-55481 ore. annktee 12, ond lek A007? 48978 grecenter@gmail.com ulna Rak Properties, House 11,Senadanga Main Rd #01940-442162| Part Three: Quantitative Reasoning 292 Chapter 14 17, Quantity A Quantity B x#0 The sum of ‘The sum of the prime the prime QuantityA Quantity B factors of 36 _ factors of 22 2. tl =i x Z®D Quantity A is greater. ® Quantity A is greater. Quantity B is greater. @D Quantity B is greater. © The two quantities ® The two quantities are equal, are equal. @® The relationship cannot ® The relationship cannot be determined from the ibe enatanined foe the information given. information given. 18. Goa badariec f 21. A7 by 24 rectangle is inscribed ina circle. What is the &- «@-p circumference of the circle? B® Quantity A is greater. @Dm Quantity B is greater. cones © The two quantities pai are equal. D 30 @® The relationship cannot be determined from the Star information given 22, Ifa <0 and b <0, what are the ‘The perimeter of a square equals possible values for athe the perimeter of a rectangle that is Indicate all that apply. not a square. 19. Quantity A Quantity B The area of The area of the square the rectangle BD Quantity A is greater. Quantity B is greater, @® The two quantities are equal. ® The relationship cannot be determined from the information given. ‘www.grecenter.net ‘Laima: Hosue 27, lock (Opp Ohanmondl Govt Boye 401921-c80840| ‘= 01720-57481 gre (=) Banani: Howse 121, Roud 4 Block A 60167740197 Anda Properties House 1, Senadanga Man Ra s01940-492162 grecenter@gmail.com w ul T v 23. In the figure, RSTU is a square with side 4. Points V and W are the midpoints of sides UT and ST, respectively. Which is the area of ARWV? @s 6 @©7 Ds es 24, What is the solution of the system of equations shown below? 6x —3y=9 4x — 3y=5 @® 22 @v @ (2) DW 12 @® (1-1 25. If x # 0, which of the following must be greater than x? Indicate al] possible choices. @ x @ x-2 x+2 @) @w-2" Bo w+2 Part Three: Quantitative Reasoning ‘Quantitative Reasoning Practice Set 293 26. A rectangle has a perimeter of 24. Which of the following could be its area? Indicate all possible choices. ® 28 B 32 © 35 DD 36 27, A square has the same area as a rectangle whose length is 2 more that its width. If the perimeter of the rectangle is 12, what is the perimeter of the square? @e @® 62 @©s @D 82 @® w 28. A cube with an edge of 3 units sits inside a cube with an edge of 5 units. How many cubic units of space are there between the smaller cube and the larger cube? cubic units 29. What is the area of a rectangle with aside of 5 and a diagonal of 132 square units wnegecenternet =o aes eis cea ean oo cle =omosomsi OLE, eae Reet as ean maennauae ‘Banani House 121,Road 4 Block A.#01677-481878

You might also like